Vous êtes sur la page 1sur 150

MOCK EXAM PHYSIOLOGY

Total number of questions: 180

SELECT THE MOST PROBABLE ANSWER


1.

A healthy young adult with BMI 23.40 lost 2 liters of fluid as sweat. He then drank same
volume of water. Which of the following would be his Dannet-Yarrow diagram, if the dotted
line represents his latest status?

A.

B.

C.

D.

E.

.
Answer E: He lost 2 liters of sweat and also drank 2 liters of water. Water loss and
gain is balanced. But when he lost 2 liters of sweat, he also lost some electrolytes
(solutes). This loss of solutes (decreased osmolarity) from ECF, makes it hyposmotic.
So fluid from ECF moves to ICF (recall definition of OSMOSIS). So ECF volume
decreases and ICF volume increases.

1 | P a g e | By Dr. Manoj Kumar Poudel (Nepal- the country where Mt. Everest is present & the country
where Buddha was born.)

2. Which of the following graph gives the relationship between alveolar ventilation in x-axis
(liter/minute) and partial pressure of carbondioxide in artery in y-axis (mm Hg)?

A.

B.

2 | P a g e | By Dr. Manoj Kumar Poudel (Nepal- the country where Mt. Everest is present & the country
where Buddha was born.)

C.

D.

E.
Answer B: This question is about alveolar ventilation equation.

3 | P a g e | By Dr. Manoj Kumar Poudel (Nepal- the country where Mt. Everest is present & the country
where Buddha was born.)

3. Which of the following can be seen in the electrocardiogram of a 24-year-old male collapsing
due to vasovagal syncope?
R-R interval
QRS complex
PR interval
A. Increased
Widened
Shortened
B. Decreased
Normal
Prolonged
C. Increased
Normal
Prolonged
D. Decreased
Widened
Normal
E. Normal
Narrow
Prolonged
Answer C: The guy collapsed because he has vasovagal syncope (A drop in cerebral
perfusion pressure due to a sudden collapse in cardiac output & vasodilation). This
happens because:
i)
Vagus nerve (PANS) slows down SA node firing [slower heart rate (BP)
increased R-R interval], slowing down of AV node conduction (prolonged
PR interval) and decreased contraction of atria only all via M-receptors AND
ii)
Withdrawal of SANS leading to slowing down of SA node [slower heart rate
(BP) increased R-R interval], slowing down of AV node conduction
(prolonged PR interval) and decreased ventricular contraction all via 1
receptors.

4 | P a g e | By Dr. Manoj Kumar Poudel (Nepal- the country where Mt. Everest is present & the country
where Buddha was born.)

4. A 39-year-old female from New Jersey, has her left sided cardiac output as 6 liter/minute. If a
drug X is injected and her left-sided cardiac output now is 12 liter/minute, what is the cardiac
output in her right side of the heart?
A. 3-5 liter/minute
B. 5-7 liter/minute
C. 7-9 liter/minute
D. 9-11 liter/minute
E. 11-13 liter/minute
Answer E: Right sided CO = Left-sided CO (systemic blood flow = pulmonary blood
flow).

5 | P a g e | By Dr. Manoj Kumar Poudel (Nepal- the country where Mt. Everest is present & the country
where Buddha was born.)

5. Lymph flow starts from this structure to be collected in small lymphatic capillaries associated
with portal triads, and from there the lymph circulates to the systemic lymphatic system finally
to be drained into venous system. Identify this structure in the histological section below:

6 | P a g e | By Dr. Manoj Kumar Poudel (Nepal- the country where Mt. Everest is present & the country
where Buddha was born.)

A.
B.
C.
D.
E.

A
B
C
D
E
Answer A: Space of Disse (peri-sinusoidal space): between a hepatocyte and
endothelium of a sinusoid. B = Kupffer cells; C = RBC; D = sinusoid; E = endothelial
cell.

6. A 52-year-old female has oligomenorrhoea. The ability of a bone in this patient to withstand
a pulling (tensile) strength is by a special substance. Which of the following factors affect the
process of synthesis/deposition of this substance in the bone?
A. Parathyroid hormone and calcitonin
B. Vitamin C and copper
C. Vitamin D and 1-hydroxylase
D. Growth factor and brain natriuretic peptide
E. Estrogen and Luetinizing hormone
Answer B: Tensile strength is given by collagen. Hydroxylation of proline & lysine
needs vitamin C and iron as co-factors. Copper is required for cross-linking of
collagen.

7. A healthy 50-year-old man who resides at sea level takes a 45-minute ride in a cable car
from near sea level to a ski resort (altitude 3050 m). Which of the following best represents
this patient 30 minutes after arrival at the resort?

7 | P a g e | By Dr. Manoj Kumar Poudel (Nepal- the country where Mt. Everest is present & the country
where Buddha was born.)

A.
B.
C.
D.
E.
F.
G.

A
B
C
D
E
F
G
Answer D: High altitude low oxygen faster breath (less CO2 in blood)
respiratory alkalosis (higher pH) compensatory excretion of HCO3- (less HCO3- in
blood).

8 | P a g e | By Dr. Manoj Kumar Poudel (Nepal- the country where Mt. Everest is present & the country
where Buddha was born.)

8. A 66-year-old patient in intensive care unit is asked to do end-inspiratory hold maneuver. The
positive end-expiratory pressure (PEEP) came out to be 5 cm Hg. His tidal volume (V T) is
maintained at 500 milliliter. If the dynamic compliance of his lung is 100 millimeter/cm Hg,
calculate the peak inspiratory pressure (PIP) by the mechanical ventilator.
A. 150 cm Hg
B. 5 cm Hg
C. 6 cm Hg
D. 10 cm Hg
E. 11 cm Hg
Answer D:
Sometimes they may ask to calculate just Lung compliance, rather than dynamic or
static.
LUNG COMPLIANCE = dV/dP (change in lung volume/change in transpulmonary pressure)
Transpulmonary pressure = Alveolar pressure Pleural pressure

There are 2 types of lung compliance: Static vs. Dynamic

Memorize this following formula too:


9 | P a g e | By Dr. Manoj Kumar Poudel (Nepal- the country where Mt. Everest is present & the country
where Buddha was born.)

9. If graph C represents normal compliance of the lungs, which of the following graphs
represents emphysema?

A.
B.
C.
D.
E.

A
B
C
D
E
Answer D: Emphysema: a form of COPD where alveoli are destroyed due to damage
in its elastic fibres. Since elastic fibres are lost, the compliance increases.

10. Following are the inferences of different laboratory values for a patient of 24 year of age.
10 | P a g e | By Dr. Manoj Kumar Poudel (Nepal- the country where Mt. Everest is present & the country
where Buddha was born.)

Serum ADH =
Serum osmolarity = 275 mOsmol/kg H2O
Urine osmolarity = 40 mOsmol/kg H2O
Urine flow rate = 5 mL/minute
Free-water clearance = positive
What is your diagnosis?
A.
B.
C.
D.
E.

Water deprivation
Syndrome of inappropriate secretion of antidiuretic hormone (SIADH)
Excessive water drinking
Nephrogenic diabetes insipidus
Central diabetes insipidus
Answer C: Excessive water drinking decrease in serum osmolarity; we need to
excrete more water, hence less ADH secreted to decrease anti-diuresis (to cause
diuresis), dilute urine, more urine flow and urine contains more free water than
osmotic water as it is dilute.

11. A 72-year-old man has half the creatinine concentration in the urine and half the urine flow
rate than the 22-year-young man has. Following are the plasma creatinine concentration of
them:
22-year-old plasma creatinine concentration = 1 mg/dL
72-year-old plasma creatinine concentration = 2 mg/dL
What percentage of GFR does the old man has with respect to the young man?
A. 6.25%
B. 12.5%
C. 25%
D. 50%
E. 100%
Answer B: Inulin is used to measure GFR because it is freely filtered, but neither
reabsorbed nor secreted. Creatinine slightly over-estimates GFR because on top of
free filtration, it is slightly secreted, but not reabsorbed, hence it can also be used to
estimate GFR.
11 | P a g e | By Dr. Manoj Kumar Poudel (Nepal- the country where Mt. Everest is present & the country
where Buddha was born.)

GFR of young man = (2a X 2b)/1 = 4ab


GFR of old man = (a X b)/2 = ab/2
% = [(ab/2)/4ab] X 100% = 12.5%
12. The murmur heard best at the 2 nd right intercostal space is systolic, crescendo-decrescendo
in nature and radiates to the carotid arteries bilaterally. The murmur increases with squatting,
decreases with standing and isometric muscular contraction. The murmur is louder during
expiration, but is also easily heard during inspiration. Identify the cause:
A. Normal
B. Aortic stenosis
C. Aortic regurgitation
D. Mitral stenois
E. Mitral valve prolapse
F. Pulmonary stenosis
G. Tricuspid regurgitation
H. Ventricular septal defect
I. Tricuspid regurgitation
J. Pulmonary valve regurgitation
Answer B:

12 | P a g e | By Dr. Manoj Kumar Poudel (Nepal- the country where Mt. Everest is present & the country
where Buddha was born.)

MURMUR OF PDA IS CONTINUOUS MURMUR (NEITHER SYSTOLIC NOR DIASTOLIC)

13 | P a g e | By Dr. Manoj Kumar Poudel (Nepal- the country where Mt. Everest is present & the country
where Buddha was born.)

13. A 2-month-old baby has 160 beats per minute heart rate. He is dyspneic and the chest x-ray
shows cardiomegaly. His pulse pressure is widened. And interestingly he has cyanosis in the
lower half of the body only. Study the following phonocardiogram which is labeled as F. If
you have correctly identified the problem, which of the following could be the most
appropriate treatment?

14 | P a g e | By Dr. Manoj Kumar Poudel (Nepal- the country where Mt. Everest is present & the country
where Buddha was born.)

A.
B.
C.
D.
E.

Indomethacin
Mitral valve replacement
Balloon valvuloplasty of pulmonary valve
Aortic valve repair surgery
Prostaglandin E1
Answer A: Patent ductus arteriosus: continuous machine-like murmur; before birth
connects pulmonary artery to aorta; should close after birth; continues to open if
prostaglandin E1 present; hence can be closed by indomethacin (NSAID) which is
cycloxigenase inhibitor, thereby decreasing prostaglandin production.

15 | P a g e | By Dr. Manoj Kumar Poudel (Nepal- the country where Mt. Everest is present & the country
where Buddha was born.)

Flow murmur: due to increased blood volume flow via valves. It occurs in anemia,
hyperthyroidism, fever, and pregnancy.
Aortic regurgitation: Wide pulse pressure; early diastolic decrescendo murmur best
heard with the diaphragm of the stethoscope when the patient is leaning forward, with
breath held at end-expiration.( increases in volume in response to maneuvers that increase afterload
eg, squatting, isometric handgrip).) and pulse as following:
Water-Hammer pulse: palpation of rapidly increasing & then
subsequently collapsing pulse.
Pistol shot pulse: auscultation finding of sharp sound over femoral artery.
14. A 36-year-old gardener is working in your garden since last 3 years. Suddenly he collapsed.
He was taken to hospital immediately (within 5 minutes). His blood pressure is found to be
80/50 mm Hg. The emergency physician prescribed intravenous normal saline,
dexamethasone and an anti-histaminics drug. Which of the following cells could have their
number increased in this patients body at this time?
A. IgA, basophils and neutrophils
B. IgE, mast cells and eosinophils
C. IgG, basophils and T-helper cells
D. IgA, neutrophils and eosinophils
E. IgE, mast cells and basophils
Answer E: Late allergic reactions vs. Early/acute allergic reactions

15. Radial pulse and femoral pulse are palpated simultaneously. An appreciable delay in the
femoral pulse is appreciated. Ankle-brachial index measurement was low at 0.8 at both legs.
What is your diagnosis?
A. Co-arctation of aorta
B. Hypervoluemia
C. Tetralogy of Fallot
D. Cervical rib
E. Persistent ductus venosus
Answer A: Radio-femoral delay of pulse due to blockage in between by co-arctation
of aorta. There is also difference in BP.

16 | P a g e | By Dr. Manoj Kumar Poudel (Nepal- the country where Mt. Everest is present & the country
where Buddha was born.)

16. A 28-year-old man with a history of intravenous drug use comes to the physician because of
a 6-week history of fever, nonproductive cough, chills, and progressive shortness of breath.
His temperature is 39C (102.2F), pulse is 110/min, respirations are 32/min and regular, and
blood pressure is 120/80 mm Hg. Physical examination shows a white, patchy, loosely
adherent exudate on the buccal mucosa bilaterally. A chest x-ray shows bilateral interstitial
infiltrates. After receiving treatment for pneumonia, he agrees to participate in a clinical study
of the effects of interleukin-2 (IL-2). After administration of IL-2, which of the following
hematologic changes is most likely in this patient?
A. Decreased CD4+ T lymphocytes
B. Decreased erythrocytes
C. Decreased platelet count
D. Increased CD4+ T lymphocytes
E. Increased erythrocytes
F. Increased platelet count
Answer D: Remember: Severe HIV means less CD4+ count and increased viral load
in blood. So, treatment will increase CD4+ count and decrease viral load in blood.
17. A 4-hour-old female newborn delivered at 28 weeks gestation has respiratory distress. Her
mother has diabetes mellitus. Steroid was given to ameliorate the respiratory problem due to
preterm delivery. Her temperature is 36.0C (96.8F), pulse is 175/min, respirations are
90/min, and blood pressure is 70/44 mm Hg. Arterial blood gas analysis on room air shows:
pH 7.16
PCO2 82 mm Hg
PO2 52 mm Hg
Endotracheal intubation and mechanical ventilation are required. The primary cause of this
patients condition is a dysfunction of which of the following cell types?
A. Dust cells
B. Clara cells
C. Pulmonary vascular endothelial cells
D. Pulmonary chondrocytes
E. Smooth muscle cells in the respiratory bronchioles
F. Type I pneumocytes
G. Type II pneumocytes
17 | P a g e | By Dr. Manoj Kumar Poudel (Nepal- the country where Mt. Everest is present & the country
where Buddha was born.)

H. Ciliated epithelium with mucociliary apparatus


Answer G: this child has respiratory acidosis due to less production of surfactant,
because of PREMATURITY.

Dont miss STEROID here.

18 | P a g e | By Dr. Manoj Kumar Poudel (Nepal- the country where Mt. Everest is present & the country
where Buddha was born.)

Law of Laplace
18. A 17-year-old single mother from Northern part of Canada has been diagnosed with tinea
corporis. She is a pure vegetarian. Which of the following nutrients most probably can be
deficient in her 5.5-month-old baby who is exclusively breast-fed by this teenage mother?
A. Calcium, protein and iron
B. Iron, vitamin D and vitamin B12
C. Sodium, calcium and potassium
D. Protein, monosodium glutamate and omega-3
E. Carbohydrate and zinc
Answer B: Iron is deficient because after 3 months, the babys storage of iron and
breast-milk lactoferritin will no longer be sufficient. Vitamin D is deficient because she
is from a place where the sun shines very less in a day; and vitamin B12 because she
is a pure vegetarian.
19. If the LV is non-compliant and atrial contraction forces blood through the AV valves, a heart
sound is produced by the blood striking the LV. Name this heart sound.
A. S1
B. S2
C. S3
D. S4
E. Thrill
F. Heave
G. Hum
H. Bruit
I. Ejection click
J. Ventricular gallop
Answer D:

19 | P a g e | By Dr. Manoj Kumar Poudel (Nepal- the country where Mt. Everest is present & the country
where Buddha was born.)

20. Calculate the approximate mean arterial pressure with the following data:
Heart rate: 70 beat per minute
Stroke volume: 70 milliliter
Total peripheral resistance: 15 mm Hg.minute/Liter
A. 50 mm Hg
B. 70 mm Hg
C. 75 mm Hg
D. 100 mm Hg
E. 120 mm Hg
Answer C:
20 | P a g e | By Dr. Manoj Kumar Poudel (Nepal- the country where Mt. Everest is present & the country
where Buddha was born.)

CO = HR X SV = 70 X 70 = 4900mL = 5liter
MAP = CO X TPR = 5 X 15 = 75 mm Hg
21. Shown below is the schematic diagram of a nephron. At which nephron site the tubular fluid
inulin concentration is the highest during high dose desmopressin therapy for a 32-year-old
patient suffering from central diabetes mellitus?

A.
B.
C.
D.
E.

A
B
C
D
E
Answer E: This is like the physiological condition in which there is high level of ADH
to conserve water, thereby producing very concentrated urine.

22. Gut commensal flora in intestine removes glucoronic acid and converts bilirubin into
urobilinogen. Which of the following vitamins are produced by intestinal commensal bacteria?
A. Vitamin A and niacin
B. Vitamin A, D, E and K
C. Vitamin C and cyanocobalamin
D. Vitamin K and biotin
E. Vitamin D and folic acid
Answer D: straight forward question
23. Heliox (a mixture of 21% oxygen and 79% helium) was used to be used in bronchial asthma
before the advent of bronchodilators. These days it is still used in the conditions where large
airways are blocked due to tumor or foreign body. It can also be used for commercial deep
diving. The reason behind these uses are because:
A. Helium has higher density but equal viscosity than that of air.
B. Helium causes more laminar flow of air and causes less resistance in the large
airways.
C. Helium has lower density and higher viscosity than that of air.
21 | P a g e | By Dr. Manoj Kumar Poudel (Nepal- the country where Mt. Everest is present & the country
where Buddha was born.)

D. Helium causes more turbulent flow of air and causes higher resistance in the small
airways.
E. Helium increases viscosity in larger airways and decreases resistance in all-sized
airways.
Answer B:

24. A 14-year-old male from slum area presented with severe diarrhea in our emergency
department. His blood pressure is 82/42 mm Hg. He is seen as hyperventilating. His lab
results are as follows:
Serum potassium: 2.9 mEq/L
Serum chloride:130 mEq/L
Which of the following could be his ABG results if his acid-base disorder is partially
compensated?

A.
B.
C.
D.
E.

7.10
7.34
7.56
7.00
7.50

27
70
26
70
46

8
39
23
16
36

Answer A:
In diarrhea the patient will have metabolic acidosis (& also hypokalemia). So, there will
be less bicarbonate ions in the plasma. This is partially corrected by respiratory
mechanism by decreasing PCO2 (by hyperventilation) in plasma. The chloride is
higher because it is replacing lost bicarbonate ions to maintain anion gap in plasma.

22 | P a g e | By Dr. Manoj Kumar Poudel (Nepal- the country where Mt. Everest is present & the country
where Buddha was born.)

25. In a 35-year-old patient, a balloon catheter is inserted and inflated at the tricuspid valve
blocking the atrio-ventricular flow. The urine output increased. Which of the following
hormones could have caused this increased urine output?
A. Anti-diuretic hormone
B. Aldosterone
C. Atrial natriuretic peptide
D. Angiotensin-II
E. Renin
Answer C: The name explains everything. It is produced by atrial cardiomyocytes. It is
secreted when atrium receives higher volume of blood. So response of ANP would be
to excrete sodium (natrium in latin). Along with sodium ions, water also gets excreted
to maintain fluid volume in the human body. Learn the following too:

26. In the graph below showing plasma hormone levels as a function of time, ovulation takes
place at which of the labeled points on the time axis?
23 | P a g e | By Dr. Manoj Kumar Poudel (Nepal- the country where Mt. Everest is present & the country
where Buddha was born.)

A.
B.
C.
D.
E.

A
B
C
D
E
Answer C: Ovulation takes place after LH surge.
Day of ovulation = cycle duration 14 days

Analyse this picture from the practice question given for Reproductive System in Week
8 & 9.
27. A drug salbutamol in nebulizer is given to a patient with acute exacerbation of bronchial
asthma. This produced an increment in diameter of the bronchus by double the size than it
was when the drug was not given. By how many times was the resistance decreased by the
drug in the bronchus?
A. 1
B. 2
C. 4
D. 8
24 | P a g e | By Dr. Manoj Kumar Poudel (Nepal- the country where Mt. Everest is present & the country
where Buddha was born.)

E. 16
Answer E: Here the question has asked the relation between radius and the
resistance. Sometimes they may ask the relation between the radius and the flow.

28. The curve below shows the percentage of the filtered load of different substances remaining
in the tubular fluid at various segments of a nephron.

Along the length of the nephron, the curve A best describes the profile of which of the
following substance/s?
A. Lysine & dextrose
B. Inulin
C. Urea
D. Para-aminohippuric acid
E. Albumin
Answer A: as glucose & amino acids are freely filtered but 100% reabsorbed in early
proximal tubule. B = Sodium; C = inulin; D = PAH
29. From the following figure of a nephron, identify the substance involved in this type of filtration,
reabsorption and excretion in kidney:

25 | P a g e | By Dr. Manoj Kumar Poudel (Nepal- the country where Mt. Everest is present & the country
where Buddha was born.)

A.
B.
C.
D.
E.
F.
G.
H.
I.
J.

Sodium
Potassium
Bicarbonate
Chloride
Calcium
Magnesium
Urea
Water
Glucose
Phosphate
Answer J: Please learn this diagram for all other ions and substances including
osmolarity and water from Renal System power-points.

30. During a study of gastric parietal cells, an investigator attempts to elicit maximum
hydrochloric acid secretion from the stomach of an experimental animal. Which of the
following combinations of substances is most likely to lead to this desired effect?

Answer B: very direct question


31. Acute obstruction of the common bile duct produced experimentally will incur which of the
following changes in plasma and urinary levels of bilirubin?

26 | P a g e | By Dr. Manoj Kumar Poudel (Nepal- the country where Mt. Everest is present & the country
where Buddha was born.)

Answer B:

32. In males inhibin is secreted by sertoli cells, whereas in females it is secreted by granulosa
cells. Increased inhibin A level, increased beta-hCG, decreased estriol and decreased alphafeto protein signifies Down syndrome in the fetus prenatally. Regarding plasma inhibin B
level:
A. It is significantly higher in fertile men than in infertile men.
B. It is significantly higher in infertile men than in fertile men.
C. It is same in concentration in fertile men and infertile men.
D. It is secreted in response to low FSH in serum and causes increased secretion of FSH
from anterior pituitary gland.
E. It has similar action to activin.

27 | P a g e | By Dr. Manoj Kumar Poudel (Nepal- the country where Mt. Everest is present & the country
where Buddha was born.)

Answer A: question is teaching you about inhibin A and B. Inhibin inhibits secretion of
FSH, when FSH level is higher in blood. Hormone activin has opposite properties to
inhibin.
Note: Plasma hCG level is very high in diseases like molar pregnancy and
choriocarcinoma.
33. In male the first sign of puberty is growth of testes & scrotal sac. Whereas which of the
following is in the order of occurrence for a normal female as she grows from child to normal
adult?
A. Thelarche pubarchemenarchegrowth spurt
B. Pubarchethelarchemenarchegrowth spurt
C. Menarchepubarchethelarchegrowth spurt
D. Growth spurtmenarchepubarchethelarche
E. Pubarchegrowth spurtthelarchemenarche
Answer A: question is teaching you the events in puberty in male vs. female.
34. A 62-year-old man presents with a 4-year progressive history of increasing lower urinary tract
symptoms. His:
Flow rate: 11 mL/sec [Normal 20-25mL/sec]
Post-void residual: 60 mL [Normal < 50mL]
Prostate volume (on transrectal ultrasonography): 65 mL [Normal < 40mL at this age]
Prostate-specific antigen (PSA) level: 3.2 ng/mL [Normal <4 ng/mL]
If the size of his prostate increases further and he gets signs and symptoms of urinary outlet
obstruction, the most common first line therapy would be:
A. 5-reductase stimulator
B. 5-reductase inhibitor
C. Aromatase stimulator
D. Aromatase inhibitor
E. 17-hydroxysteroid hydroxylase stimulator
F. 17-hydroxysteroid hydroxylase inhibitor
Answer B: PSA increases in prostatic cancer only. This case is about urinary outlet
obstruction from bladder due to enlarged prostate in male. Prostate growth is due to
DHT (dihydrotesterone). Testosterone is converted to DHT by 5-alpha reductase.
Hence 5 alpha reductase inhibitors like finisteride slows growth of prostate.
35. In the provided graph with serum concentration of LH and rate of testosterone synthesis,
which of the letters represent 21-year-old male taking large doses of methyl testosterone

28 | P a g e | By Dr. Manoj Kumar Poudel (Nepal- the country where Mt. Everest is present & the country
where Buddha was born.)

A.
B.
C.
D.
E.

A
B
C
D
E
Answer D:

36. Following is the inference of different hormones level in a patient of 24 year of age. What is
your diagnosis?
Sex steroid:
LH: 1.95 U/L
FSH: 2.30 U/L
A. Primary hypogonadism
B. Pituitary hypogonadism
C. Post-menopausal state due to early menopause
D. Anabolic steroid therapy (male)
E. Inhibin infusion (male)
F. GnRH infusion (at constant rate)
G. GnRH infusion (in a pulsatile manner)
Answer B:

29 | P a g e | By Dr. Manoj Kumar Poudel (Nepal- the country where Mt. Everest is present & the country
where Buddha was born.)

37. This is hormone X: has biologic activity similar to LH; and "informs" corpus luteum that
fertilization has occurred. Which of the following curves best approximates it level?

A.
B.
C.
D.
E.

A
B
C
D
E
Answer A:

30 | P a g e | By Dr. Manoj Kumar Poudel (Nepal- the country where Mt. Everest is present & the country
where Buddha was born.)

After delivery, all hormones level goes back to pre-pregnant state, EXCEPT prolactin.
38. In pregnant woman, lactation is inhibited. Although higher level of estrogen stimulates
prolactin secretion, estrogen also down regulates prolactin receptors in the mammary glands.
But, why does lactation provides natural contraception?
A. Prolactin level comes back to pre-pregnant state
B. Prolactin completely atrophies ovaries
C. Dopamine increases prolactin secretion
D. TRH decreases prolactin secretion
E. Prolactin inhibit GnRH
Answer E: high prolactin high milk secretion (GALACTORRHOEA) + there is no
menstrual period, as prolactin suppresses ovulation by inhibiting synthesis & release of GnRH
(INFERTILITY).

39. Eunuchs are tall because


A. Estrogens stimulate the growth of long bones
B. Excess LH delays epiphyseal closure of the long bones
C. Reduced androgen and estrogen delays epiphyseal closure in long bones
D. The lack of testes stimulates closure of the epiphyses
E. They secrete excess androgen
Answer C: Eunuch have lesser sex hormones. One of the functions of sex hormones
(androgens & estrogen) is growth and delayed epiphyseal plate fusion.

31 | P a g e | By Dr. Manoj Kumar Poudel (Nepal- the country where Mt. Everest is present & the country
where Buddha was born.)

40. A 25-year-old woman at 32 weeks' gestation begins taking a drug to delay the onset of
preterm labor. After the first dose, she notices tremulousness in her hands. Which of the
following types of receptors is most likely to be involved in this effect?
A. 1-Adrenergic
B. 2-Adrenergic
C. Dopaminergic (D1)
D. Muscarinic (M1)
E. NMDA
Answer B: Tocolytics (2-agonist, atosiban, etc.): are antagonist to oxytocin receptor; and are
used to prevent premature labour.

41. During the second and third trimesters of pregnancy, women have increased levels of
circulating glucocorticoids but have few of the catabolic responses associated with excess
glucocorticoids. The most likely explanation of this finding is increased:
A. autoantibodies to steroids
B. metabolic clearance of steroids
C. steroid-binding proteins
D. steroid receptors
E. steroid uptake mechanisms
Answer C: In pregnancyIncrease in plasma ACTH, total cortisol and transcortin
(corticosteroid binding protein) in pregnancy. Due to parallel increase in ACTH increase
secretion of cortisol and its binding protein, the free cortisol is constant. It is produced by liver
and secretion is regulated by estrogen.

42. Following are the photomicrographs obtained from a 23-year-old female patient. The biopsy
sample is from the same organ but at two different days of a month. Which of the following
hormones is primarily responsible for inducing this change (from photomicrograph on your
left to photomicrograph on your right) in appearance?

A. Thyroxine
B. Progesterone
32 | P a g e | By Dr. Manoj Kumar Poudel (Nepal- the country where Mt. Everest is present & the country
where Buddha was born.)

C. Estrogen
D. Corticosterone
E. Aldosterone
Answer B: Progesterone is higher during secretory/luteal phase of menstruation.
Picture on your left: Photomicrograph of straight uterine glands in deep endometrium
during proliferative phase. Smooth muscle of myometrium is also seen. H&E stain. Medium
magnification.
Picture on your right: Photomicrograph of uterine glands. During the luteal phase, the
uterine glands become tortuous and their lumen is filled with secretions. Some edema is
present in connective tissue. H&E stain. Medium magnification.

43. A 25-year-old patient was brought by his friends as he had a lacerated wound on the lateral
surface of right foot. The doctor decided to put few stitches. So injected a local anesthetic
drugs around the wound. The drug, lignocaine, would work by:
A. Inhibiting voltage-gated sodium channels in the cell membrane of neurons.
B. Inhibiting ligand-gated sodium channels in the post-synaptic membrane.
C. Inhibiting voltage-gated calcium channels in the pre-synaptic membrane.
D. Inhibiting release of neurotransmitters from presynaptic membrane.
E. Inhibiting dorsal root ganglion.
Answer A: Remember local anesthetics are weak base (BH + or R-NH3+).

33 | P a g e | By Dr. Manoj Kumar Poudel (Nepal- the country where Mt. Everest is present & the country
where Buddha was born.)

44. A 28-year-old female presented with headache and restlessness for 2 years and vague
abdominal pain for 6 months. Investigations revealed a mass below the lower pole of the
right kidney on ultrasonography. An attempt at removal of the mass was attended by severe
hypertensive crisis mandating abandonment of the procedure. Patient was referred to our
center for further management. On admission, her blood pressure was 220/120 mm Hg. 24hour urinary vanillylmandelic acid was 9.50 milligram (normal range 26-121 microgram).
Computerized tomography abdomen revealed a 5-cm diameter mass below the lower edge
of the right kidney near the midline with solid and cystic component and enhancement on
post-contrast study. In this case "pure" -blocker must never be used due to the risk of such
treatment leading to unopposed -agonism to prevent:
A. Rebound hypertension
B. Tumor metastasis
C. Essential hypertension
D. Volume overload
E. Profound hypotension
Answer A: haha, the explanation is in the question itself.

34 | P a g e | By Dr. Manoj Kumar Poudel (Nepal- the country where Mt. Everest is present & the country
where Buddha was born.)

Know the precursors, synthesis and process of termination of action of important


neurotransmitters.
45. A 36-year-old female who presented with muscle wasting, moon face and buffalo hump also
has high blood glucose level, striae on her abdomen & thigh and osteoporosis seen on x-ray.
35 | P a g e | By Dr. Manoj Kumar Poudel (Nepal- the country where Mt. Everest is present & the country
where Buddha was born.)

Her blood pressure on both arms in sitting position after 20 minutes of rest is 155/105 mm of
Hg. 24-hour urine free cortisol is found to be elevated. Which of the following do you expect
further?
A. Hyperkalemia
B. Hypoglycemia
C. Hypotension
D. Virilization and menstrual disorders
E. Anorexia and weight loss
Answer D: This is the case where adrenal cortex in hyper-secreting all of its
hormones.

46. A 48-year-old patient ingested many tablets of salicyclic acid. The doctor believed that his
patient ingested it to the level of toxicity. The patient has acidosis. Which of the following is
caused by acidosis?
A. Decreased ionized calcium in blood
B. Increased ionized calcium in blood
C. Increased albumin bound calcium in blood
D. Hypoventilation
E. Vasoconstriction
Answer B: A case of metabolic acidosis due to ingestion of excessive acid.

36 | P a g e | By Dr. Manoj Kumar Poudel (Nepal- the country where Mt. Everest is present & the country
where Buddha was born.)

47. Following are the laboratory finding of a thin and diaphoretic female patient who presented to
our Out-Patient Department with tremor, exophthalmos, palpitations, weakness and heat
intolerance:
Serum TSH: 0.1 mU/L
Serum T3:10 nmol/L
Serum T4: 210 nmol/L
Serum TSI: 51 mU/L (Noraml lab value < 10 mU/L)
A. Hashimotos thyroiditis
B. Toxic nodular goiter
C. Iodine deficiency goiter
D. Papillary carcinoma of thyroid
E. Graves disease
F. Cretinism
G. TRH-secreting tumor
H. Levothyroxine therapy
I. Surgery for hyperthyroidism
J. Pituitary adenoma
Answer E:

37 | P a g e | By Dr. Manoj Kumar Poudel (Nepal- the country where Mt. Everest is present & the country
where Buddha was born.)

48. Study the following histological section. Which of the following labeled structures secretes a
hormone whose function is listed below?
i. Sodium and water retention via
principal cells in late distal
tubules and collecting duct.
ii. Secretion
of
hydrogen
and
potassium ions via alphaintercalated cells and principal
cells.

A.
B.
C.
D.

A
B
C
D

38 | P a g e | By Dr. Manoj Kumar Poudel (Nepal- the country where Mt. Everest is present & the country
where Buddha was born.)

E. E
Answer B: A (capsule); B (glomerulosa); C (fasciculata); D (reticularis) & E (medulla)
The action given in the question is about aldosterone.

49. Which of the following best characterizes the pulmonary circulation?

Answer E: direct answer question.


50. A patient lying on his back and breathing normally has a mean left atrial pressure of 7 cm
H2O; a mean pulmonary arterial pressure of 15 cm H 2O; a cardiac output of 4 L/min; and an
anteroposterior chest depth of 15 cm, measured at the xiphoid process. Most of his lung is
perfused under which of the following conditions?
A. Zone 1
B. Zone 2
C. Zone 3
D. Zone 4
Answer C: The question explains so that the conclusion is Pa > PV > PA. Hence it is
zone 3.

39 | P a g e | By Dr. Manoj Kumar Poudel (Nepal- the country where Mt. Everest is present & the country
where Buddha was born.)

51. Which of the following best characterizes alveolar ventilation and blood flow at the base,
compared with the apex, of the lungs of a healthy standing person?

Answer A:

40 | P a g e | By Dr. Manoj Kumar Poudel (Nepal- the country where Mt. Everest is present & the country
where Buddha was born.)

52. Regional differences in ventilation perfusion ratios affect gas tensions in the pulmonary
blood. Which of the following best describes the gas tensions in the blood leaving the
alveolar capillaries of a healthy standing individual?

Answer C: For explanation refer to the chart in question 51.


53. A patient suffering from smoke inhalation injury is mechanically ventilated. His condition is
stable but he is completely paralyzed and relies on accurate ventilator settings to maintain
his blood gas status. His respiratory parameters are;
Tidal volume = 500 ml
Respiratory rate = 15/min
FiO2 (fractional O2 percentage that is being added to the delivered air) = 40%
PEEP = 8 cm H2O
I:E = 1.02 (Inspiratory is to expiratory duration ratio)
An arterial sample is drawn and produces following result:
pH 7.54
pCO2 35 mm Hg
pO2 90 mm Hg
HCO3- 25 mEq/L
ASSUMING HE HAS A SIMPLE DISTURBANCE which of following alterations in respiratory
parameter will correct his disturbance:
A. decrease FiO2
B. decrease PEEP
C. decrease respiratory rate
D. increase FiO2
E. increase tidal volume
Answer C: This patient has respiratory alkalosis, which can be managed by
decreasing the exhalation of carbondioxide. This can be done by decreasing
respiratory rate.

41 | P a g e | By Dr. Manoj Kumar Poudel (Nepal- the country where Mt. Everest is present & the country
where Buddha was born.)

54. A 35-year-old man has a vital capacity (VC) of 5 L, a tidal volume (TV) of 0.5 L, an inspiratory
capacity of 3.5 L, and a functional residual capacity (FRC) of 2.5 L. What is his expiratory
reserve volume (ERV)?
A. 4.5 L
B. 3.9 L
C. 3.6 L
D. 3.0 L
E. 2.5 L
F. 2.0 L
G. 1.5 L
Answer G:
VC = ERV + (TV + IRV)
IC = IRV + TV
So ERV = VC - IC = 5 - 3.5 = 1.5 L

42 | P a g e | By Dr. Manoj Kumar Poudel (Nepal- the country where Mt. Everest is present & the country
where Buddha was born.)

55. A man who has tidal volume of 550 mL is breathing at a rate of 14 breaths/minute. The PCO 2
in his arterial blood is 40 mm Hg, and PCO2 in his expired air is 30 mm Hg. Calculate his
minute ventilation and alveolar ventilation.
A. 7700 mL/minute and 5768 mL/minute respectively.
B. 7700 mL/minute and 138 mL/minute respectively.
C. 138 mL/minute and 5768 mL/minute respectively.
D. 412 mL/minute and 138 mL/minute respectively
Answer A:

43 | P a g e | By Dr. Manoj Kumar Poudel (Nepal- the country where Mt. Everest is present & the country
where Buddha was born.)

56. A 48-year-old homemaker woman is admitted to the cardiology ward. Her BMI is 39. She told
that she has to wake up few times in the middle of the night as she could not breathe during
the sleep since last 3 years. Her blood pressure is 156/112 mm Hg on both arms. Her EKG
shows arrhythmia. What could be her next finding?
A. Left ventricular hypertrophy
B. Right ventricular hypertrophy
C. Hypoxic vasodilation in the lungs vasculature
D. No snoring
E. During sleep periodic arterial oxygen saturation increases.
Answer B: Due to less oxygen there is hypoxic vasoconstriction in pulmonary
circulation. This causes right ventricle to work more to push the blood in lungs for
oxygenation. In a long run the RV hypertrophies. In hypertrophy the size of cell
increase, it is due to increase in number of organelles inside the cell.

57. Following are the inferences from the laboratory findings of a 20-year-old male baseball
player who presented in our emergency room with respiratory distress:
Left shift of oxygen-hemoglogin dissociation curve
Hemoglobin concentration: 15.5 g/dL
Plasma haptoglobin: 100 mg/dL (Normal: 45-165 mg/dL)
Mean corpuscular volume: 90 fL
Oxygen content of blood:
Reticulocyte count: 1%
Hematocrit: 46%
Cytochrome C oxidase: functioning normally
A. Decreased cardiac output
B. Intravascular hemolytic anemia
C. Megaloblastic anemia
D. Cyanide poisoning
E. Carbonmonoxide poisoning
Answer E:

44 | P a g e | By Dr. Manoj Kumar Poudel (Nepal- the country where Mt. Everest is present & the country
where Buddha was born.)

45 | P a g e | By Dr. Manoj Kumar Poudel (Nepal- the country where Mt. Everest is present & the country
where Buddha was born.)

58. In response to mild-moderate exercise in a normal healthy young adult of 70 kilogram in


weight, which of the following will occur?
A. Decreased oxygen consumption
B. Decreased CO2 production
C. Decreased ventilation rate
D. Increased arterial PO2 and PCO2
E. Metabolic alkalosis
F. Decreased venous PCO2
G. Decreased pulmonary blood flow
H. Decrease in physiological dead space
I. Oxygen-hemoglobin dissociation curve shifts to left
J. V/Q ratio not evenly distributed in the lungs.
Answer H:

46 | P a g e | By Dr. Manoj Kumar Poudel (Nepal- the country where Mt. Everest is present & the country
where Buddha was born.)

59. A 32-year-old woman with pulmonary fibrosis is most likely to minimize the work of breathing
by using which of the following breathing patterns?
Respiratory Frequency
Tidal Volume
A.

B.

normal
C.

D.
Normal

E.
Normal
normal
F.
Normal

G.

H.

normal
Answer G: Pulmonary fibrosis is restrictive lung disease. Hyperventilation at rest and
exercise is caused by the reflexes arising from the lungs and the need to maintain minute
ventilation by reducing tidal volume and increasing respiratory frequency.

60. A 64-year-old male has been smoking 2 packets of cigarettes a day for last 20 years. His
chest is barrel shaped. Following are the finding of his respiratory functions:
Total lung capacity:
FEV1:
Forced vital capacity:
Peak flow:
Functional residual volume:
Residual volume:
Which of the following is true about FEV1/FVC in this case?
47 | P a g e | By Dr. Manoj Kumar Poudel (Nepal- the country where Mt. Everest is present & the country
where Buddha was born.)

A.
B.
C.
D.
E.

Increased
Normal
Decreased
Can be either increased or normal
At first it decreases and then remains increased.
Answer C:

61. A 46-year-old female has left-sided heart failure. There is impaired gas exchange in her
lungs, thats why she is dyspneic. She is diaphoretic, has anxiety and orthopneic. Her
respiratory rate is 32 per minute. Which of the following receptors might have worked to
increase her respiratory rate in this case?
A. Irritant receptors
B. J-receptors
C. Joint receptors
D. Muscle receptors
E. Stretch receptors
Answer B:

48 | P a g e | By Dr. Manoj Kumar Poudel (Nepal- the country where Mt. Everest is present & the country
where Buddha was born.)

62. Peripheral chemoreceptors detect O2, CO2 and hydrogen ions, whereas central
chemoreceptors detect:
A. Interstitial hydrogen ions directly
B. Arterial CO2 directly
C. Arterial O2 directly
D. Arterial bicarbonate ions indirectly
E. Water content in intracellular and extracellular space
Answer A:

49 | P a g e | By Dr. Manoj Kumar Poudel (Nepal- the country where Mt. Everest is present & the country
where Buddha was born.)

63. A 1-day-old neonate died due to pulmonary embolism. The A-a oxygen difference was very
very high. Which of the following would be her V/Q ration just before her death?
A. Infinite
B. High
C. Low
D. Zero
Answer A:

50 | P a g e | By Dr. Manoj Kumar Poudel (Nepal- the country where Mt. Everest is present & the country
where Buddha was born.)

64. Which of the following is in the correct path of CO2 from the tissue to the atmosphere?
A. Reaction with H2O to make H2CO3, dissociation to H+ and HCO3-, H+ combines with
imidazole side chain of hemoglobin, carried back to lungs as HHb + and HCO3-, reverse
reaction forms CO2.
B. O2 is metabolized to CO2, reaction with H2O to make H2CO3, H2CO3 combines with
imidazole side chain of hemoglobin, H2CO3Hb+ is carried back to the lungs, reverse
reaction forms CO2.
C. Reaction with H2O to make H2CO3, dissociation to H+ and HCO3-, HCO3- combines with
imidazole side chain of hemoglobin, carried back to the lungs as HCO 3-Hb+ and H+,
reverse reaction forms CO2.
D. O2 is metabolized to CO2, reaction with H2O to make H2CO3, dissociation to H+ and
HCO3-, carried back to lungs in this form, reverse reaction forms CO 2.
Answer A:

51 | P a g e | By Dr. Manoj Kumar Poudel (Nepal- the country where Mt. Everest is present & the country
where Buddha was born.)

65. Study the following diagram. The solid curve shifted to the place where the dotted curve is.
Which of the following could be the cause in plasma?

A.
B.
C.
D.
E.

Decreased PCO2
Increased hydrogen ion concentration
Decreased temperature
Decreased 2,3-DPG
Carbonmonoxide poisoning
Answer B:

52 | P a g e | By Dr. Manoj Kumar Poudel (Nepal- the country where Mt. Everest is present & the country
where Buddha was born.)

66. Mr. Smiths urine flow rate is 1.5 mL/minute. If the osmotic water is 0.75 mL/minute, what is
the free water clearance in this patient?
A. 1.5 mL/minute
B. 0.75 mL/minute
C. 2.25 mL/minute
D. 0 mL/minute
E. 1 mL/minute
Answer B:

67. Total gas concentration in solution is equal to the total sum of dissolved gas, bound gas and
chemically modified gas. Which of the following is responsible for partial pressure of the gas
in the blood?
A. All dissolved gas, bound gas and chemically modified gas.
B. Dissolved gas
C. Chemically modified gas
D. Bound gas
Answer B: direct answer question.
68. Alveoli A has double the thickness of the membrane than the alveoli B. The surface area
of alveoli B is 2 times more than that of alveoli A. Partial pressure gradient of oxygen is 60
mm Hg and diffusion coefficient of oxygen is 20 times less than that of carbondioxide. What
is the ratio of volume of oxygen transferred across alveoli A to alveoli B?
A. 1:4
B. 1:2
C. 1:1
D. 2:1
E. 4:1
Answer A:

53 | P a g e | By Dr. Manoj Kumar Poudel (Nepal- the country where Mt. Everest is present & the country
where Buddha was born.)

69. The following data are obtained from a patient:


Inulin clearance = 170 liter/day
Plasma HCO3- concentration = 25 millimole/liter
Urine HCO3- concentration = 0 millimole/liter
Urine pH = 5.8
Titrable acid in urine = 26 millimole/day
Ammonium ion in urine = 48 millimole/day
Calculate the total amount of H+ secreted by the kidneys:
A.
B.
C.
D.
E.
F.

4250 millimole/day
26 millimole/day
71 millimole/day
74 millimole/day
4324 millimole/day
0 millimole/day
Answer E:

54 | P a g e | By Dr. Manoj Kumar Poudel (Nepal- the country where Mt. Everest is present & the country
where Buddha was born.)

Remember this formula too:

70. The following graph shows the concentration of various substances at different segments of
the nephron. The top brown line belongs to creatinine as labeled. Which of the following
curves denote the fate of water (without ADH) in the nephron?

A. A
55 | P a g e | By Dr. Manoj Kumar Poudel (Nepal- the country where Mt. Everest is present & the country
where Buddha was born.)

B.
C.
D.
E.

B
C
D
E
Answer D:

71. Regarding filtration, reabsorption, secretion and clearance, which of the following is true?
Uricosuric interference Creatinine/penicillin Inulin
Ionised weak acid
A.
Reabsorption
Secretion
Filtration
Cleared
B.
Cleared
Filtration
Secreted
Reabsorption
C.
Filtration
Cleared
Reabsorbed
Secreted
D.
Secreted
Reabsorped
Cleared
Filtration
Answer A: direct question. Please memorize the answer.
72. A 32-year-old woman decides to donate a kidney to his sister, who is in the end-stage kidney
disease stage. After human leukocyte antigen typing it was found that she would be a
suitable donor. She is admitted to the hospital, and her left kidney is removed and
transplanted into her sister. Which of the following would be expected to be decreased in the
donor after full recovery from the operation?
A. Creatinine clearance
B. Creatinine production
C. Daily excretion of sodium
D. Plasma creatinine concentration
E. Renal excretion of creatinine
Answer A:

56 | P a g e | By Dr. Manoj Kumar Poudel (Nepal- the country where Mt. Everest is present & the country
where Buddha was born.)

73. Mrs. Rani, 38-year-old female, gets recurrent calcium oxalate multiple stones in her bilateral
ureters and kidneys. She complains of bone pain and has depression. Shown below is her
current EKG:

Which of the following drugs may be used to treat this condition?


A. Thiazide diuretics
B. Potassium sparing diuretics
C. Loop diuretics
D. Acetazolamide
E. Osmotic diuresis
Answer C: This is a case of hypercalcemia (dont confuse EKG with hyperkalemia,
which only has tall, tented T-wave). Hypercalcemia has short QT duration and wide Twave. Loop diuretics act in thick ascending limb against Na+ 2Cl- K+ cotransporter.
57 | P a g e | By Dr. Manoj Kumar Poudel (Nepal- the country where Mt. Everest is present & the country
where Buddha was born.)

Hence lumen positive potential is not generated. This is prevents reabsorption of


positively charged ions like calcium and magnesium.

Other points to remember:


Thiazide diuretics decrease risk of forming calcium containing stone.
Alkalinization of urine prevents uric acid stone.
Ingestion of a high-oxalate diet, Ingestion of a high-protein diet, Restriction of
fluid intake increase chances of renal stone formation.
74. A 45-year-old man visited a GP yesterday after his son figured out that he has high blood
pressure measured at home. The GP told, Don't drink coffee or smoke cigarettes for 30
minutes prior to the measurement. Go to the restroom and relax for 5-10 minutes before the
measurement.After this the GP found out his blood pressure to be 165/115 mmHg at 2
different times. So, after detailed history and examination, the GP ordered several
investigations and prescribed a combination of anti-hypertensive drugs. On top of antihypertensive property, for one of the drugs prescribed, which of the following would cause
decreased secretion of K+ by the distal tubule?
Plasma renin activity: 0.1 ng/mL/hr (Normal: 0.2 to 4 ng/mL/hr)
A. Loop diuretics
B. Thiazide diuretics
C. Uricosurics
D. Osmotic diuresis
E. Aldosterone antagonists
Answer E: Spironolactone is an example of aldosterone antagonist (K+ sparing
diuretics). This can be used to treat hypertension in patient who has lower plasma
renin.
Functions of aldosterone: sodium ion retention water retention; secretion of
potassium and hydrogen ions.
75. Given the following data, calculate the glomerular filtration rate (GFR) of this patient:
Plasma creatinine = 0.90 milligram/100 milliliter
58 | P a g e | By Dr. Manoj Kumar Poudel (Nepal- the country where Mt. Everest is present & the country
where Buddha was born.)

Plasma inulin concentration = 1.8 milligram/100 milliliter


Plasma urea = 14.4 milligram/100 milliliter
24-hour urine volume = 1,440 milliliter (=1.0 milliliter/minute)
Urine concentration of urea = 144 milligram/100 milliliter
Urine concentration of inulin = 180 milligram/100 milliliter
Urine concentration of creatinine = 100 milligram/100 milliliter
A.
B.
C.
D.
E.
F.

10 milliliter/minute
100 milliliter/minute
111 milliliter/minute
125 milliliter/minute
130 milliliter/minute
154 milliliter/minute
Answer B: Inulin measure GFR the best, as it is only filtered; neither secreted nor
reabsorbed. Creatinine also measures GFR, but slightly over-estimates GFR, as it is
filtered and slightly secreted too. Here INULIN concentration would be the best to
calculate GFR by the following formula:

76. Following are the laboratory values of different investigations on a 35-year-old patient after
injection of PAH and inulin. Calculate the effective renal blood flow:
Urine flow rate: 1 mL/min
Plasma PAH concentration: 1 mg%
Plasma inulin concentration: 4 mg%
Urine PAH concentration: 600 mg%
Urine inulin concentration: 400 mg%
Hematocrit: 50%
A. 200 mL/min
B. 300 mL/min
C. 800 mL/min
D. 1091 mL/min
E. 1200 mL/min
Answer E: Effective plasma flow can be measured by PAH, as it is freely filtered and
secreted. By using following formula:

59 | P a g e | By Dr. Manoj Kumar Poudel (Nepal- the country where Mt. Everest is present & the country
where Buddha was born.)

.
Then with the help of plasma volume and hematocrit, the blood volume can be calculated by
following formula:

77. Following is the data about 32-year-old male patient from Chhangchhangdi, Nepal.
Renal plasma flow = 500mL/minute (Normal 660mL/minute)
Glomerular filtration rate (GFR) = 150mL/minute (Normal 120mL/minute)
Filtration fraction = 40% (Normal 20%)
What is the cause?
A.
B.
C.
D.
E.

A 2 cm ureterolithiasis
Malnutrition
Dehydration
Constriction of afferent arteriole
Constriction of efferent arteriole
Answer E:

60 | P a g e | By Dr. Manoj Kumar Poudel (Nepal- the country where Mt. Everest is present & the country
where Buddha was born.)

78. Which of the following can occur with chronic renal failure (chronic kidney disease) in a 55year-old mentally disturbed patient from Alaska?
A. Low blood pressure
B. Low serum urea and creatinine
C. Hypokalemia
D. Hypercalcemia
E. Metabolic alkalosis
F. Volume contraction
G. Highly responsive renin angiotensin-II aldosterone system (RAAS)
H. Buffalo hump
I. Hypophosphatemia
J. Anemia
Answer J: kidney secretes erythropoietin. So, when kidney is damaged there is less
secretion of erythropoietin and less erythropoiesis in red bone marrow anemia.

79. A normal healthy adult females glomerular filtration rate (GFR) is been studied for a research
purpose. The concentration of plasma inulin is kept constant with intravenous infusion. The
GFR was found out to 120 milliliter/minute at 1:00pm. Then she was given one liter of water
61 | P a g e | By Dr. Manoj Kumar Poudel (Nepal- the country where Mt. Everest is present & the country
where Buddha was born.)

to drink. She drank it and after some time her GFR was measured again. Which of the
following could be true about her new GFR?
A. Lower than 120 milliliter/minute
B. Higher than 120 milliliter/minute
C. Equal to 120 milliliter/minute
D. Filtration equilibrium occurs everywhere.
E. Any of the above can happen because she is a beautiful healthy adult.
Answer C:

80. A 49-year-old male from Arkansas received his urine and plasma electrolytes laboratory
results:
Plasma Na+ concentration: 142 mEq/L
Plasma K+ concentration: 2.1 mEq/L
Plasma Cl- concentration: 98 mEq/L
Urine Na+ concentration: 60 mEq/L
Urine K+ concentration: 55 mEq/L
Urine osmolarity: 520 mOsmol/L
What is your diagnosis?
A. Pituitary damage
B. Acute renal failure
C. Water intoxication
D. Primary hyperaldosteronism
E. Normal findings
62 | P a g e | By Dr. Manoj Kumar Poudel (Nepal- the country where Mt. Everest is present & the country
where Buddha was born.)

Answer D:

81. Following is the dyselectrolytemic EKG of a 72-year-old patient who is a known case of
chronic renal failure.

10 milliliter 10% calcium gluconate is prescribed intravenously over 10 minutes duration in


the emergency room under EKG-monitoring. What does this drug do in this case?
A. Decrease cardiac excitability.
B. Lower the deranged electrolyte temporarily.
C. Increase the elimination of the deranged electrolyte via urine.
D. Decrease the excretion of the deranged electrolyte in the kidneys.
E. Decrease reabsorption from distal tubule.
63 | P a g e | By Dr. Manoj Kumar Poudel (Nepal- the country where Mt. Everest is present & the country
where Buddha was born.)

Answer A: The ECG show tall, tented T-wave signifying hyperkalemia. Look at the
following chart for treatment of hyperkalemia:

82. The oncotic pressure of blood is 25 mm Hg and interstitial fluid pressure is 5 mm Hg. The
hydrostatic pressure in capillary is 20 mm Hg and interstitial fluid oncotic pressure is 2 mm
Hg. Which of the following statement would be true?
A. Filtration occurs
B. Absorption occurs
C. Edema formation occurs
D. Hydraulic conductance does not determine the property of fluid movement.
E. No net fluid movement.
Answer B:

83. A 4-year-old male kid has been getting recurrent urinary tract infections (acute
pyelonephritis). The infection responds well to common antibiotics, but the concern is to find
out the cause of the recurrence. The urologist ordered micturating cystourethrogram. The
radiographer inserted the catheter into the bladder via external urethral opening and filled the
bladder with radiocontrast material. After taking the catheter out, the radiologist asked the kid
64 | P a g e | By Dr. Manoj Kumar Poudel (Nepal- the country where Mt. Everest is present & the country
where Buddha was born.)

to void. During the voiding, the radiologist appreciated radio-contrast material in the ureter.
What is your diagnosis?
A. Vesicoureteral reflux
B. Bladder polyps
C. Retrograde ejaculation
D. Neurogenic bladder dysfunction
E. External urethral sphincter anomaly
Answer A: this is a simple logical question
Retrograde ejaculation: semen instead of getting ejaculated into urethra, goes to
urinary bladder; a cause of infertility; occurs when there is damage in the sphincter of
bladder special after prostate surgery; drugs used to treat HTN, behign enlargement of
prostate (Benign prostatic hypertrophy), etc., nerve injury due to spinal trauma,
multiple sclerosis, DM, etc.
84. A 63-year-old man underwent a radical prostatectomy surgery. He has a stable bladder but
by Valsalva maneuver at Pves 30 cm of H2O, he has a problem which if shown with arrows
in the pressure-flow graph of his urinary bladder. What is the diagnosis?

A.
B.
C.
D.
E.

Post-voidal dribbling
Overflow incontinence
Urge incontinence
Neurogenic bladder dysfunction
Stress incontinence
Answer E: The arrow shows incontinence during Valsalva maneuver. Hence it is
stress incontinence.

65 | P a g e | By Dr. Manoj Kumar Poudel (Nepal- the country where Mt. Everest is present & the country
where Buddha was born.)

66 | P a g e | By Dr. Manoj Kumar Poudel (Nepal- the country where Mt. Everest is present & the country
where Buddha was born.)

85. Which of the following tests best distinguishes central diabetes insipidus from nephrogenic
diabetes insipidus?
A. Injection of hypertonic saline
B. Injection of insulin
C. Injection of ADH (vasopressin)
D. Water deprivation
E. Water loading
Answer C:
67 | P a g e | By Dr. Manoj Kumar Poudel (Nepal- the country where Mt. Everest is present & the country
where Buddha was born.)

Vasopressin test: Nephrogenic DI can be distinguished from central DI by


administration of desmopressin, the synthetic analog of ADH. Lack of a prompt
decrease in urine volume and an increase in urine osmolarity within 2 hours after
injection of desmopressin is strongly suggestive of nephrogenic DI.

Best initial diagnostic test: water deprivation test (it will differentiate between polydipsia and other
two diabetes insipidus (central and nephrogenic).
Most accurate test: Vasopressin/desmopressin stimulation test (it will differentiate between
central vs nephrogenic DI).

86. In a 32-year-old patient, the glomerular filtration rate (GFR) at 9 am was 90 mL/minute. The
physician gave an injection intravenously. Now (9 pm) the GFR came out to be
120mL/minute. Which of the following can cause increase in GFR?
A. An decrease in renal flow
B. An increase in hydraulic pressure in Bowmans capsule
C. An increase in glomerular capillary oncotic pressure.
D. Vasodilation of afferent arteriole
E. Vasodilatation of efferent arteriole.
Answer D: this is a direct question. GFR increases by increasing blood flow in kidney
and decreasing blood outflow from kidney. GFR depends on the factors in Starling
pressure.
87. To calculate ankle-brachial pressure index for a 43-year old female patient, blood pressures
at different parts of the body were measured. Blood pressure in the upper arm (brachial
artery) came out to be 120/80 mm Hg and blood pressure in the lower limb (posterior tibial
artery) came out to be 180/90 mm Hg. With these results, what is your interpretation?
A. Normal findings
B. Deep vein thrombosis
C. Blood vessel hardening due to peripheral vascular disease
D. Dorsal pedis artery rupture
E. Abnormal effect of gravity
68 | P a g e | By Dr. Manoj Kumar Poudel (Nepal- the country where Mt. Everest is present & the country
where Buddha was born.)

Answer C: ABI compares the arteries in upper limb vs. lower limb to find out
pathology in between them.

88. A 24-year-old man got a deep cut wound in his thigh and his friend who is a 4th year medical
student estimated that he had lost about 1000mL of blood. On examination his heart rate is
78 beats per minute, his pulse pressure has decreased due to an increase in diastolic blood
pressure, respiratory rate is 24 per minute and he looks pale. Which of the following would
cause an immediate physiological response in this patient?
A. Renin secretion due to stimulation of alpha-1 receptors
B. Vasoconstriction due to stimulation of sympathetic nervous system
C. Vasodilatation due to stimulation of parasympathetic nervous system
D. ANP secretion due to stimulation of stretch receptors in the heart
E. Histamine release from mast cells

Answer B: 1000 mL is around 25% blood loss. So it falls in class II hemorrhage where
there is generalized vasoconstriction. And the question is asking immediate
physiological response. Immediate response is by SANS, whereas renin is secreted
by stimulation of beta-1 receptors as a slower response (RAAS stimulation) to
hemorrhage.

69 | P a g e | By Dr. Manoj Kumar Poudel (Nepal- the country where Mt. Everest is present & the country
where Buddha was born.)

70 | P a g e | By Dr. Manoj Kumar Poudel (Nepal- the country where Mt. Everest is present & the country
where Buddha was born.)

89. Shown below are 2 different EKGs labeled as A and B. Based on the voltage and duration
of electrical activity on the graphs identify the conditions A and B.

A
71 | P a g e | By Dr. Manoj Kumar Poudel (Nepal- the country where Mt. Everest is present & the country
where Buddha was born.)

B
A.
B.
C.
D.
E.

Atrial flutter and ventricular fibrillation respectively.


Atrial fibrillation and paroxysmal supraventricular tachycardia respectively.
Premature ventricular contractions and dextrocardia.
Wolff-Parkinson-Wolff syndrome and ventricular tachycardia.
3rd degree heart block and Left bundle branch block.
Answer A: Atril flutter has saw-tooth shaped EKG. Ventricular fibrillation has
quiverings.

90. A substance was given via intravenous route to a 29-year-old bisexual patient at 6 pm by the
nurse in Tapoin Hospital, Castries. This substance can have following properties:
Can cause transient heart block by acting on AV node.
Can cause endothelial-dependent smooth muscle relaxation in normal segment of
arteries in heart.
Can be used to identify rhythm and treat for AVRT and AVNRT.
Causes bronchoconstriction as side effect.
Identify the IV injected substance:
A. Dopamine
B. Epinephrine
72 | P a g e | By Dr. Manoj Kumar Poudel (Nepal- the country where Mt. Everest is present & the country
where Buddha was born.)

C.
D.
E.
F.
G.
H.
I.
J.
K.

Serotonin
Adenosine
Endorphin
Enkephalin
Substance P
Glycine
Glutamate
GABA
Neuropeptide Y
Answer D:

91. In the following figure, where does the mitral valve continuously remain open?
73 | P a g e | By Dr. Manoj Kumar Poudel (Nepal- the country where Mt. Everest is present & the country
where Buddha was born.)

A.
B.
C.
D.
E.
F.
G.
H.

1
2
3
4
41
12
23
34
Answer E: the following diagram is very important.

Cardiac work: The work of the left ventricle can also be thought of as the area within
the pressure-volume loop.
Cardiac efficiency: The more the cardiac work area shifts to the right, the lesser the
efficiency of the heart. And it is vice-versa if the area shifts to left.
92. This question has 2 parts to answer:

74 | P a g e | By Dr. Manoj Kumar Poudel (Nepal- the country where Mt. Everest is present & the country
where Buddha was born.)

2000 daltons does not, however, when a fluorescent molecule of 500 daltons is injected into
a single cell of an epithelial layer, it quickly appears in the adjacent cells. Which of the
following is responsible for the spread of the smaller molecule?
Rapid efflux of newly secreted bile from canaliculi into hepatic interstitium can be caused by
structural disruption or functional alteration of which of the following components of normal
interactions between hepatocytes?
A. Adherent junctions and zona occudens respectively
B. Integrins and laminins respectively
C. Desmosomes and hemidesmosomes respectively
D. Desmins and dyeinins
E. Gap junctions and tight junctions respectively
Answer E: Functions of gap junction and tight junction. Dont miss that
hemidesmosome connects the extracellular matrix with integrin (transmembrane
protein of the cell membrane). Revise all the intermediate filaments:

93. A 2-year-old Caucasian male with a history of recurrent pulmonary infections and weight loss
has a sweat test positive. Regarding his recurrent pulmonary infections, which of the
following ion-transfer mechanisms and which of the following type is the dysfunctional
transmembrane protein respectively in this case?
A. cAMP activated chloride secretion and ATP-gated
B. Ca2+ activated chloride uptake and ATP-gated
C. cAMP-activated chloride uptake and G-protein gated
D. cGMP-stimulated chloride secretion and G-protein gated
E. Tyrosine kinase activated chloride reuptake and Ca2+-gated
75 | P a g e | By Dr. Manoj Kumar Poudel (Nepal- the country where Mt. Everest is present & the country
where Buddha was born.)

F. cAMP-stimulated chloride uptake and ATP-gated


Answer A:

Note the chloride channel differences in the direction of action in sweat glands and
rest of body secretions. Dont miss the SWEAT TEST.
94. The figure on the left shows ventricular and an aortic pressure tracing curve during a single
cardiac cycle. The figure on the right represents left ventricular pressure-volume loop for the
same cardiac cycle. Which of the following letters in pressure tracing curve matches the
number 5 in pressure-volume loop?

A.
B.
C.
D.

A
B
C
D

76 | P a g e | By Dr. Manoj Kumar Poudel (Nepal- the country where Mt. Everest is present & the country
where Buddha was born.)

E.
F.
G.
H.
I.

E
F
G
H
I
Answer C:

77 | P a g e | By Dr. Manoj Kumar Poudel (Nepal- the country where Mt. Everest is present & the country
where Buddha was born.)

95. The following is the combined graph of vascular and cardiac function graphs. The dotted line
is the new condition after a change occurred in the patients body. Which of the following
could be the best reason for this new change?

A.
B.
C.
D.

Increased total peripheral resistance


Increased blood volume
Positive inotropism
Decreased total peripheral resistance

78 | P a g e | By Dr. Manoj Kumar Poudel (Nepal- the country where Mt. Everest is present & the country
where Buddha was born.)

E. Decreased blood volume


Answer D:

79 | P a g e | By Dr. Manoj Kumar Poudel (Nepal- the country where Mt. Everest is present & the country
where Buddha was born.)

96. Look at the graph below carefully. The dotted line is new change in pressure volume curve
for left ventricle. The new change could most probably due to:

A.
B.
C.
D.
E.

Increased preload
Increased afterload
Increased contractility
Decreased work load to cardiac muscles
None of the above
Answer A:

80 | P a g e | By Dr. Manoj Kumar Poudel (Nepal- the country where Mt. Everest is present & the country
where Buddha was born.)

97. The following is a normal ECG. Which of the following could be the Q-wave due to?

81 | P a g e | By Dr. Manoj Kumar Poudel (Nepal- the country where Mt. Everest is present & the country
where Buddha was born.)

A.
B.
C.
D.
E.
F.

Atrial depolarization
Atrial repolarization
Septal depolarization
Ventricular depolarization
Ventricular repolarization
Papillary muscle depolarization
Answer C:

In the meantime, please learn which ECG segment corresponds to which segment in
the action potential of ventricle below:

82 | P a g e | By Dr. Manoj Kumar Poudel (Nepal- the country where Mt. Everest is present & the country
where Buddha was born.)

98. Which of the following options is true regarding the properties of components of
cardiovascular system listed? [CO = cardiac output]
Highest compliance
Highest resistance Receives 100% of CO Lowest velocity
A.
Veins
Arterioles
Lungs
Capillaries
B.
Arterioles
Aorta
Skeletal muscle
Veins
C.
Aorta
Capillaries
Brain
Arteries
D.
Capillaries
Venules
Heart muscle
Venules
E. Elastic arteries
Muscular arteries
Kidneys
Aorta
Answer A: direct answer question.
83 | P a g e | By Dr. Manoj Kumar Poudel (Nepal- the country where Mt. Everest is present & the country
where Buddha was born.)

Highest capacitance/compliance: veins


Highest amount of blood in CVS: veins
Highest resistance: arterioles (highest pressure fall occurs here n it determines amount of
blood flow)
Pressure: [artery > vein]; [systemic > pulmonary]
Right-sided cardiac output is equal to left-sided cardiac output.

99. Following figure shows the systemic arterial pressure during the cardiac cycle. The "blip" in
the arterial pressure curve, called the dicrotic notch (or incisura), is due to:

A.
B.
C.
D.
E.

Closure of mitral valve


Closure of aortic valve
Closure of pulmonary valve
Closure of tricuspid valve
Opening of mitral valve
Answer B: During the systole aortic valve opens, so that blood from LV goes out to
aorta. Just before diastole, the aortic valve should close.

100.
The end-systolic volume of a cardiac cycle of a 32 year old male from Cidar Heights is
70 milliliters. His heart rate is 70 beats per minute. If his ejection fraction were 0.50, what
would be his cardiac output?
A. 35 milliliters
B. 70 milliliters
C. 140 milliliters
D. 4900 milliliters
E. 9800 milliliters
Answer D:

84 | P a g e | By Dr. Manoj Kumar Poudel (Nepal- the country where Mt. Everest is present & the country
where Buddha was born.)

So End-diastolic volume = 140

So SV = 140 70 = 70

CO = 70 X 70 = 4900mL
101.

Find the right choice:


Chemoreceptors
Baroreceptors
A. Carotid sinus
Carotid body
B. Carotid body
Carotid sinus
C. Carotid sinus
Carotid body
D. Carotid body
Carotid sinus

Local metabolites least control in blood flow


Skeletal muscle during exercise
Skin
Skeletal muscle during rest
Heart, brain, lungs, kidneys

Answer B:

85 | P a g e | By Dr. Manoj Kumar Poudel (Nepal- the country where Mt. Everest is present & the country
where Buddha was born.)

102.
The contractility of heart muscle increased thereby increasing cardiac output in a
patient who is volunteering for a clinical research, after a known drug was injected
intravenously when it was used as a control. Which of the following is related physiologically
to this process?
A. Gs
B. Gi
C. Gq
D. It is ligand-gated.
E. Phospholipase C
F. Receptor tyrosine kinase
Answer A: Remember QIS mneumonic. It is asking about heart positive inotropism,
hence beta receptor. If it were skeletal muscle it would be ligand-gated. For tyrosine
kinase, they are insulin, growth factor and cytokines.
103.
When a person moves from supine position to standing position, which of the
following compensatory changes occurs?
A. Increased PR interval
B. Decreased cardiac output
C. Decreased total peripheral resistance
D. Increased contractility
E. Decreased heart rate
Answer D:

86 | P a g e | By Dr. Manoj Kumar Poudel (Nepal- the country where Mt. Everest is present & the country
where Buddha was born.)

104.
Look at the graph below. Cardiac muscle normally operates only on the ascending
limb of length-tension curve because cardiac muscles than skeletal
muslces.

A.
B.
C.
D.
E.
F.

Faster myosin-ATPase
Uses more calmodulin
Are stiffer
Slower myosin-ATPase
Uses less calmodulin
Less stiffer
Answer C: direct question.

105.
A 48-year-old man has been treated with digitoxin. Digitoxin is positive inotropic drug
obtained from plant foxglove. Which of the following does it inhibit directly to bring about
positive inotropic effect?
A. Sodium-calcium exchanger
B. Sodium-potassium pump
C. Calcium-hydrogen pump
D. L-type calcium channel
E. T-type calcium channel
Answer B:

106.
If pulmonary veins have the lowest pressure in the circulation, which of the following
variables is most similar in pulmonary circulation and systemic circulation?
87 | P a g e | By Dr. Manoj Kumar Poudel (Nepal- the country where Mt. Everest is present & the country
where Buddha was born.)

A.
B.
C.
D.
E.
F.
G.

Diastolic arterial pressure


Blood flow
Venous pressure
Capillary pressure
Resistance to flow
Mean arterial pressure
Systolic arterial pressure
Answer B: direct question. Dont miss concept of Pulmonary Capillary Wedge
Pressure too.

107.

Which of the following is FALSE regarding cardiac electrophysiology?


Atrial contraction precedes ventricular contraction.
In ventricles, contraction starts from apex to base.
Phase 4 depolarization in SA node is due to fast sodium channels.
The calcium channels used in plateau of ventricular action potential and upstroke of
SA nodal action potential have different properties.
E. Plateau phase is found in action potential of ventricles, atria and Purkinjie systems.
F. Septum depolarizes from left to right.
A.
B.
C.
D.

Answer C: You will seldom get negative questions in NBME Shelf exam. This is just
to learn 5 more points. Hope you learnt.
108.
if:

AV node is a latent pacemaker. It would predominate its role in the cardiac contraction
A.
B.
C.
D.
E.

Its action potential is of longer duration.


SA nodal action potential is of shorter duration than that of AV node.
Bundle of His intrinsic firing rate becomes lesser than that of AV node.
Purkinjie fibres intrinsic firing rate becomes lesser than that of AV node.
Its intrinsic firing rate is higher than that of SA node.
Answer E: this is direct question.

109.
A.
B.
C.
D.
E.
F.
G.

Which of the following is true in terms of their location:


Micturition center anterior pituitary gland
Pneumotaxic center medulla
Respiratory center anterior hypothalamus
Vasomotor center pons
Vomiting center midbrain
Thermoregulation center anterior thalamus
Cardiac center Medulla
Answer G:

88 | P a g e | By Dr. Manoj Kumar Poudel (Nepal- the country where Mt. Everest is present & the country
where Buddha was born.)

110.
There is an increase in normal body temperature when a person has fever. There is
also an increase in normal body temperature when a person has hyperthermia. But fever
and hyperthermia has very different patho-physiology. Regarding these, which of the
following is TRUE?
A. The set-point temperature in hyperthermia is higher.
B. The set-point temperature in fever is higher.
C. The temperature regulation mechanism is abnormal in fever.
D. The temperature regulation mechanism is functioning normally in hyperthermia.
E. Inhalational anesthetic cannot cause malignant hyperthermia.
Answer B:

89 | P a g e | By Dr. Manoj Kumar Poudel (Nepal- the country where Mt. Everest is present & the country
where Buddha was born.)

111.
We report 24 cases with lung adenocarcinoma treated with Drug X who developed
bilateral lower extremity edema and erythema, a condition rarely described. In all cases,
symptoms resolved with discontinuation or dose reduction of the drug. Which of the following
can probably cause edema in this case?
A. Heart failure
B. Venous dilation
C. Extracellular fluid contraction
D. Hyperalbuminemia
E. Hyperthermia
Answer A:

90 | P a g e | By Dr. Manoj Kumar Poudel (Nepal- the country where Mt. Everest is present & the country
where Buddha was born.)

112.
A 45-year-old patient is under sulfonyl urea drug, glipizide to treat diabetes mellitus
type 2. He is from wealthy family and his body mass index is 35. Renal function test came
out to be normal. In this patient, another drug, propranolol (1 antagonist) is contraindicated
(or can prove fatal) because:
A. Propranolol can increase hepatic gluconeogenesis.
B. Propranolol can decrease production of insulin.
C. Propranolol can mask hypoglycemia.
D. Propranolol can decrease the bioavailability of glipizide.
E. Propranolol is a competitive inhibitor on the same receptor where glipizide binds.
Answer C:

113.
A 64-year-old diabetic patient from Salt Lake has poor control over his blood sugar
level despite treatment with 500 mg metformin administered twice a day. He had come to
surgery out-patient department with a foul-smelling wound on his right foot. On examination
the tissue looks gangrenous. Which of the following could be the reason for blood supply
compromise in that region due to the chronic effect of uncontrolled hyperglycemia?
A. Inhibition of nNOS
B. Inhibition of IP3/DAG
C. Inhibition of formation of oxygen radicals
D. Inhibition of eNOS
E. Inhibition of protein kinase C
Answer D:

91 | P a g e | By Dr. Manoj Kumar Poudel (Nepal- the country where Mt. Everest is present & the country
where Buddha was born.)

92 | P a g e | By Dr. Manoj Kumar Poudel (Nepal- the country where Mt. Everest is present & the country
where Buddha was born.)

114.
A 5-year-old girl falls through the ice while skating on an outdoor pond. She is
removed from the water within 1 minute, but dry clothing is not available, and she is still cold
and wet 20 minutes later. Which of the following mechanisms helps maintain the patients
core temperature during the period following her rescue?
A. Cutaneous vasodilation
B. Diving response
C. Increased thermoregulatory set point
D. Release of endogenous pyrogen
E. Shivering
Answer E:
93 | P a g e | By Dr. Manoj Kumar Poudel (Nepal- the country where Mt. Everest is present & the country
where Buddha was born.)

94 | P a g e | By Dr. Manoj Kumar Poudel (Nepal- the country where Mt. Everest is present & the country
where Buddha was born.)

115.
A 24-year-old laborer is working in the hot sun in Riyadh, Saudi Arab. Which of the
following would be true in his body physiology?
Rectal temperature
Cutaneous blood vessels
A.
Normal
Normal
B. Decreased
Constricted
C. Decreased
Dilated
D.
Increased
Constricted
E.
Increased
Dilated
Answer E: Body temperature will increase, so body tries to lose heat by cutaneous
vasodilation after increased arterio-venous shunting in the skin.
116.
A 23-year-old woman is scheduled to undergo surgical repair of an aortic
malformation. A ganglionic blocking agent is administered before the procedure to decrease
her blood pressure. The hypotension is achieved by binding of the drug at which of the
following receptors?
A. Muscarinic
B. Nicotinic
C. 1-Adrenergic
D. 1-Adrenergic
E. 2-Adrenergic
Answer B:

117.
A 22-year-old woman is participating in a study that measures renal function.
Evaluation of serum and urine shows the following:
Serum
Urine
95 | P a g e | By Dr. Manoj Kumar Poudel (Nepal- the country where Mt. Everest is present & the country
where Buddha was born.)

Na+ (mEq/L)
140
100
+
K (mEq/L)
4
50
Glucose (mg/dL)
100
0
Creatinine (mg/dL)
1
100
PAH (mg/mL)
0.1
50
Assuming a urine flow of 1 mL/min, which of the following best represents her filtration
fraction?
A. 0.10
B. 0.20
C. 0.30
D. 0.40
E. 0.50
Answer B:
GFR here is found from creatinine (properties very near to inulin) = 100 X 1/1 = 100
mL/min
Renal plasma flow = PAH clearance = 50 X 1/0.1 = 500
Filtration fraction = GFR/RPF = 100/500 = 0.20

118.
A hormone is known to activate phospholipase C with subsequent release of calcium
from internal stores. The release of calcium most likely occurs as a result of an increase in
the concentration of which of the following intracellular second messengers?
A. Calcium
B. cAMP
C. cGMP
D. Diacylglycerol
E. Inositol 1,4,5-trisphosphate
Answer E:
96 | P a g e | By Dr. Manoj Kumar Poudel (Nepal- the country where Mt. Everest is present & the country
where Buddha was born.)

119.
A 28-year-old woman is brought to the emergency department after a motor vehicle
accident in which her chest struck the steering wheel. Pulse is 125/min and blood pressure
is 90/60 mm Hg. She is diaphoretic and has dyspnea on exertion. Head and neck veins are
distended. Heart sounds are distant and there are no murmurs. The radial pulse pressure
decreases abnormally during inspiration. The most likely diagnosis is:
A. cardiac tamponade
B. hemorrhage from a ruptured descending aorta
C. mitral valve prolapse
D. pulmonary embolism
E. rupture of a papillary muscle
Answer A: Cardiac tamponade: compression of heart due to accumulation of fluid in
pericardial space.
120.
An asymptomatic 42-year-old man with essential hypertension is started on an
angiotensin-converting enzyme inhibitor. He started having cough after starting the drug.
Which of the following shows the most likely effect of treatment on circulating levels of
aldosterone, renin, and bradykinin?
Aldosterone/Renin/Bradykinin
A.

B.

C.

D.

no change
E.

F.

Answer C: Cough is due to bradykinin. Aldosterone is less as ACE is inhibited. Renin is


more due to feedback mechanism of less aldosterone. Bradykinin causes cough.

97 | P a g e | By Dr. Manoj Kumar Poudel (Nepal- the country where Mt. Everest is present & the country
where Buddha was born.)

121.
A series of experiments is performed to determine the mechanism by which a
pharmacologic agent traverses cell membranes and accumulates within target cells. The
rate of transport depends on the concentration of the drug only. When the extracellular
concentration of the agent exceeds 10 mM, no further increase in the rate of uptake is
observed. Structurally similar compounds pass through the cell membrane, but at a lower
rate. Ouabain, an inhibitor of Na+-K+-ATPase, fails to inhibit transport. Which of the following
is the most likely mechanism by which this agent enters cells?
A. Antiport
B. Ion-gated coupling
C. Facilitated diffusion
D. Simple diffusion
E. Symport
Answer C: It is diffusion because rate of transport depends only on concentration of the drug.
The process is saturable after 10mM, so this must not be simple diffusion. It is facilitated
diffusion. The other favouring reason is competitive inhibitors. But it is not active transport
because Ouabain (inhibitor of Na+-K+ ATPase) fails to inhibit transport.

122.
During contraction of striated muscle, the thick and thin filaments alter their
relationship with each other and the sarcomere shortens. Which of the following bands is
most likely to maintain a constant length during muscle contraction?
A. A
B. H
C. I
D. M
98 | P a g e | By Dr. Manoj Kumar Poudel (Nepal- the country where Mt. Everest is present & the country
where Buddha was born.)

Answer A:

123.
The graph shows an isovolumetric pressure-flow curve obtained at 25% of total lung
capacity. An increase in which of the following is most likely to increase the plateau value of
the expiratory flow rate?

A.
B.
C.
D.
E.
F.

Airway resistance
Amount of surfactant in the alveolar space
Ciliary beating frequency
Expiratory effort
Lung volume
Parasympathetic activity of fibers innervating the airways
Answer E: Peak flow rate primarily reflects large airway flow and depends on the voluntary
effort and muscular strength of the patient. For high flow we need less resistance. Review
Poiseulles equation.

124.
A 72-year-old woman comes to the physician because of an episode of acute
substernal chest pain that occurred while she was rushing to catch a bus. She has basilar
crackles at both lung bases. S1 is normal and S2 is decreased. A grade 3/6 systolic murmur
99 | P a g e | By Dr. Manoj Kumar Poudel (Nepal- the country where Mt. Everest is present & the country
where Buddha was born.)

is heard best at the upper right sternal border and radiates to the neck. The following data
obtained during cardiac catheterization were taken before and during exercise:

While exercising, the patient has shortness of breath but no chest pain. Which of the
following is the most likely cause of the shortness of breath?
A. Decreased cardiac output during exercise
B. Decreased pulmonary blood flow
C. Increased aortic pressure
D. Increased end-diastolic pressure of the left ventricle during exercise
E. Tachycardia
Answer D: basilar crackles at rest, systolic murmur radiating to neckand chest pain with
exercise is most consistent with late aortic stenosis. Although if patient was having crackles at
rest, in aortic stenosis the BP during exercise would be expected to drop or stay constant due
to outflow obstruction and reduced systemic resistance. In any case, if left ventricle end
diastolic volume (LVEDV) is increased in exercise, this means that the left heart preload is
increased, preload ~ volume, which tells you that D must be right regardless of the underlying
process, which is most likely aortic stenosis.
A is not answer because according to the chart cardiac output has not decreased during
exercise.
B is not answerbecause systemic and pulmonary blood flow must be equal over any
appreciable length of time.
C and E are not answer because they wont cause shortness of breath by themselves.

125.
During which phase of the ventricular action potential shown below, does
norepinephrine influence the influx of Ca 2+?

A.
B.
C.
D.
E.

A
B
C
D
E

100 | P a g e | By Dr. Manoj Kumar Poudel (Nepal- the country where Mt. Everest is present & the country
where Buddha was born.)

Answer C:

126.
A 20-year-old man ingests 1 L of water over 1 hour. Which of the following sets of
changes is most likely at the end of the hour?

A.
B.
C.
D.
E.
F.
G.
H.

Answer E: simple logical question.


127.
During a marathon, a 28-year-old woman initially has pain and distress but suddenly
feels exhilaration about halfway through the race. The best explanation is most likely to be
increased release of.
A. ACTH
B. -endorphin
C. epinephrine
D. glucagon
E. somatostatin
Answer B: Endorphin is opioid peptides in CNS for pain modulation. It is secreted as byproduct when ACTH is synthesized to stimulate secretion of cortisol during the stress. If there
is increased ACTH synthesis, the patient will have hyperpigmentation as the byproducts of
ACTH have MSH which stimulates melanin synthesis.

128.
Following is the picture of a dead man in the autopsy room. Observe the phenomenon
in its right upper limb. This process occurs because of:
101 | P a g e | By Dr. Manoj Kumar Poudel (Nepal- the country where Mt. Everest is present & the country
where Buddha was born.)

A. Absence of ATP hydrolysis which is necessary to generate energy needed for


relaxation of skeletal muscle
B. Absence of ATPase to hydrolyse the ATP to form ADP & Pi.
C. Old actin-myosin bridges detach only after the myosin binds a new ATP molecule.
D. Calcium ions that continue to get accumulated in high concentration inside the muscle
cells even after death.
E. Linking of calcium ion and calmodulin.
Answer C: direct question.
129.
During cardiac catheterization in a normal person, the blood sample withdrawn from
the catheter shows 60% oxygen saturation, and the pressure recording shows oscillations
from a maximum of 26 to a minimum of 14 mmHg. The catheter tip was most likely located in
the:
A. Ductus ateriosus
B. Foramen ovale
C. Left atrium
D. Pulmonary artery
E. Right atrium
Answer D: This vessel has deoxygenated blood. Pressure profile can be found
below:

102 | P a g e | By Dr. Manoj Kumar Poudel (Nepal- the country where Mt. Everest is present & the country
where Buddha was born.)

130.
A.
B.
C.
D.
E.

Under resting conditions, the arterio-venous O2 difference is greatest in the:


Brain
Heart muscle
Kidney
Skeletal muscle
Skin

A.
B.
C.
D.
E.

Answer B: the organ which needs to work more has more arterio-venous O2
difference. Here heart is the one.
Salivary secretion:
contains the arteriolar vasoconstrictor (bradykinin) and R-protein.
has a low volume relative to the weight of the glands producing it.
is increased by both sympathetic and parasympathetic stimulation.
is inhibited during nausea.
is modified by the secretion of the Na+ during its passage through the ducts.

131.

Answer C: direct question.


132.
A.
B.
C.
D.
E.

Secretion of PTH is increased by:


An increase in extracellular concentration of phosphate
An increase in extracellular concentration of magnesium
High intake of dietary calcium
Hypocalciuria
Treatment with thiazide diuretics

Answer A: we know the secretion of PTH is stimulated by less calcium ion


concentration in plasma. But dont miss that secretion of PTH is stimulated by an
increase in phosphate too.
133.
In humans, total adrenalectomy is invariably fatal without replacement therapy
whereas hypophysectomy is not. This is because, after hypophysectomy,
A. Adrenal catecholamines compensate for the metabolic actions of cortisol
103 | P a g e | By Dr. Manoj Kumar Poudel (Nepal- the country where Mt. Everest is present & the country
where Buddha was born.)

B.
C.
D.
E.

Plasma concentrations of angiotensin II do not increase to values that are toxic


The adrenal cortex undergoes compensatory hypertrophy
The secretion of aldosterone is not markedly decreased
Tissue requirement for corticosteroids fall to low levels
Answer D: Regulation of aldosterone secretion is more dependent on RAAS than
ACTH.

134.
The graph shows tracings of acid output from the gastric fistula and bicarbonate
output from the pancreatic fistula of a fasting dog during continuous IV infusion of gastrin. At
the arrow (X), a peptide is injected in. The peptide is most likely:

A.
B.
C.
D.
E.

ADH (vasopressin)
Cholecystokinin
Motilin
Secretin
Somatostatin
Answer D: The injected peptide decreased secretion of acid from stomach and increased
secretion of bicarbonate from pancreas. This peptide must be SECRETIN.

104 | P a g e | By Dr. Manoj Kumar Poudel (Nepal- the country where Mt. Everest is present & the country
where Buddha was born.)

135.
Solutions X and Y are separated by a semipermeable membrane impermeable to the
solute. The concentration of solutes at time zero is represented by the dots:

At equilibrium, the volumes of compartments X and Y will be:

A.

B.

C.
105 | P a g e | By Dr. Manoj Kumar Poudel (Nepal- the country where Mt. Everest is present & the country
where Buddha was born.)

D.

E.
Answer B: Here water moves from an area of lower concentration of solute to an area of
higher concentration of solute. Note the exact amount of fluid level decreased or increased as
well.

136.
The recordings shown were made by inserting a microelectrode into SA cells.
Recording X was made before exposing the heart to a particular agent, and recording Y was
made after adding the agent. The two recordings are superimposed for ease of comparison.

During the portion of the recordings indicated by M, the permeability to which of the following
ions decreases?
A. Ca2+
B. ClC. HCO3D. K+
E. Na+
Answer D: For potassium, it is only permeable during phase phase 3. Chloride and
bicarbonate are not involved in SA node potential. Calcium ions permeability increase in
phase 0, i.e. upstroke. Funny sodium channel and to some extent calcium channel are in
phase 4.

106 | P a g e | By Dr. Manoj Kumar Poudel (Nepal- the country where Mt. Everest is present & the country
where Buddha was born.)

137.
Transport of D-glucose and L-glucose across the luminal membrane of an epithelial
cell proceeds at similar rate by which of the following processes?
A. Cotransport
B. Countertransport
C. Facilitated diffusion
D. Primary active transport
E. Simple diffusion
Answer E: In simple diffusion, their rate is similar, because it is due to concentration gradient
only. The answer is not facilitated diffusion, because the carrier protein has higher affinity to
D-glucose than L-glucose, so rate is not same in this latter case of facilitated diffusion (rate of
D-glucose will be higher than that of L-glucose via facilitated diffusion; glucose mainly gets
transported by this process). Dont forget that protein carriers can go saturation making the
curve flat at transport maximum. Study the picture below and answer 4 subsequent questions
for revision:

107 | P a g e | By Dr. Manoj Kumar Poudel (Nepal- the country where Mt. Everest is present & the country
where Buddha was born.)

Answer can be either a single option or the mul

138.
Membrane potential trajectory if membrane permeability increases for both sodium
and potassium ions:

A. A
B. B
C. C
108 | P a g e | By Dr. Manoj Kumar Poudel (Nepal- the country where Mt. Everest is present & the country
where Buddha was born.)

D. D
E. E
Answer B:

139.
Which curve best illustrates the volume caused by immersion of the cell into an
aqueous solution of 100 mmol/L of CaCl2?

A.
B.
C.
D.
E.

A
B
C
D
E

109 | P a g e | By Dr. Manoj Kumar Poudel (Nepal- the country where Mt. Everest is present & the country
where Buddha was born.)

Answer C: 100 mmol/L of CaCl2 = 300 mOsmol (we multiplied by 3 because 1


molecule of calcium chloride has 3 ions, and hence its vant Hoff factor is 3), which is
iso-osmotic to ICF and ECF both. Hence no fluid movement, and no change in the
size of the cell.
140.
Exposure to the sun in hot desert environment for a prolonged period of time without
adequate intake of fluids would produce:

Answer B: In dehydration, we lose hypotonic solution.


141.

In the diagram below the shift from X to Y can be caused by:

A.
B.
C.
D.
E.

Changes in afterload
Changes in preload
Changes in myosin ATPase activity
Changes in number of active cross-bridges
Spatial summation of fibers
Answer C: With the same force there is increased initial velocity of contraction. It is
due to changes in myosin ATPase activity. This gives difference between skeletal
muscle and smooth muscle.

142.
The figure below depicts isometric length-tension relationship of skeletal muscle.
Identify the region where there is least overlapping between actin and myosin:
110 | P a g e | By Dr. Manoj Kumar Poudel (Nepal- the country where Mt. Everest is present & the country
where Buddha was born.)

A.
B.
C.
D.
E.

A
B
C
D
E
Answer E: Please remember that the best overlapping is at C.

143.
A 48 year old drunkard got a blunt force injury at the base of his neck on the left side
during a fight with another drunkard in the Island Breeze bar. On top of local soft tissue
injury, he also had some problems in his face. He has a partial ptosis on his left eye,
anhidrosis on the left side of the face and enophthalmos all on the left side. What could be
his next possible clinical feature regarding his trauma?
A. Flaring of nostrils
B. Miosis
C. Excessive sneezing
D. Drooling of saliva from left corner of his mouth
E. Loss of Glabellar reflex.
F. Fasciculations on left upper eyelid
Answer B: ipsilateral, when sympathetic chain is damaged.

111 | P a g e | By Dr. Manoj Kumar Poudel (Nepal- the country where Mt. Everest is present & the country
where Buddha was born.)

144.
true?
A.
B.
C.
D.
E.
F.
G.
H.
I.
J.

If you electrically stimulate parasympathetic system, which of the following would be


Bronchoconsriction
Ejaculation
Mydriasis
Positive inotropism
Vascular smooth muscle constriction in skin and sphlannic area.
Vascular smooth muscle dilation in skeletal muscles.
Decreased salivary secretion.
Decreased peristalsis
Detrusor muscle relaxation
Formation of goose humps
Answer A: The chart below is very important.

112 | P a g e | By Dr. Manoj Kumar Poudel (Nepal- the country where Mt. Everest is present & the country
where Buddha was born.)

Notes:
PANS in the heart does not have supply to ventricles (only to atria).
SANS supply of sweat gland is muscarinic (cholinergic).
SANS supply of adrenal gland is nitotinic (cholinergic).
In salivary gland stimulation for secretion is PANS > SANS. (both increase secretion)

145.
This disease is an autoimmune disorder, caused by auto-antibodies against
presynaptic voltage-gated calcium channels in the NMJ leading to muscle weakness. It is
mostly associated with small cell lung cancer. Name the disease:
A. Lambert-Eaton syndrome
B. Organophosphorus poisoning
C. Muscular dystrophy
D. Myasthenia gravis
113 | P a g e | By Dr. Manoj Kumar Poudel (Nepal- the country where Mt. Everest is present & the country
where Buddha was born.)

E. Tetanus
F. Alzheimer's disease
Answer A: Lambert Eton Syndrome is highly associated with Small/Oat Cell Lung
Cancer.

In the meantime, learn this too: Cocaine acts as a serotoninnorepinephrine


dopamine reuptake inhibitor, also known as a triple reuptake inhibitor (TRI).
146.
A 24-year-old female from low socio-economic family visits our out-patient
department. Her symptoms, signs and laboratory investigation values are consistent with
vitamin B12 insufficiency. Which of the following is directly linked with this condition if the
cyanocobalamin insufficiency is due to pancreatic insufficiency?
A. Crohns disease
B. Bacterial overgrowth
C. Caecal resection
D. R protein
E. Haptoglobin
Answer D: R-protein (haptocorrin) is secreted by salivary glands. It binds with vitamin
B12 and forms a complex to protect Vit B12 from damage by gastric acid in stomach.
As soon as this complex reaches duodenum, pancreatic protease cleaves the
complex. Then the vit B12 binds with intrinsic factor (secreted by parietal cells in
stomach) for it absorption in ileum. Hence we can get Vit B12 deficiency if we have
pancreatic insufficiency.
147.
Parasomnias such as night terrors, nocturnal enuresis, sleepwalking, and somniloquy
occur in which of the following stages of normal sleep?
A. Stage 1 and 2
B. Stage 2 and 3
C. Stage 3 and 4
114 | P a g e | By Dr. Manoj Kumar Poudel (Nepal- the country where Mt. Everest is present & the country
where Buddha was born.)

D. Stage 4 and REM sleep


E. REM sleep
Answer C: Please consult SLeEP power-point slides.
148.
At 10 years of age, a pediatrics patient from Marigot bay still tolerates only limited
amounts of milk. The defect in severe familial infantile lactose intolerance seems to be
localized in the gastric mucosa. Acquired lactase deficiency can appear later in childhood in
this syndrome. If lactase deficiency appears in this patient in future, what kind of diarrhea will
he most probably suffer from?
A. Osmotic diarrhea
B. Secretory diarrhea
C. Motility-related diarrhoea
D. Inflammatory diarrhoea
E. Bloody diarrhea
Answer A: If we have a disease which causes hyper-peristalsis we get motility-related
diarrhea. Drug loperamide is anti-motility agent.

115 | P a g e | By Dr. Manoj Kumar Poudel (Nepal- the country where Mt. Everest is present & the country
where Buddha was born.)

149.
A.
B.
C.
D.
E.

Ascorbic acid is a potent enhancer of iron absorption because it:


Enhances the absorption of heme iron.
Enhances the activity of heme oxygenase .
Is a reducing agent, thereby helping to keep iron in the ferrous state.
Decreases the production of ferritin by enterocytes.
Stimulates production of transferring.
Answer C: this is a direct question.
Note, iron gets absorbed in ferrous form, but gets transported in plasma in ferric form.
Iron needed for oxygen binding is in ferrous form. Methemoglobin (doesnt bind to
oxygen) is hemoglobin with iron in ferric state. Carboxyhemoglobin is hemoglobin
which bound carbonmonoxide. Carbaminohemoglobin is hemoglobin which has
bound carbondioxide.
Fetal hemoglobin (HbF) has higher affinity for oxygen then adult hemoglobin (HbA).

150.
Oesophageal manometry is being done. Manometer is at certain portion of
gastrointestinal gract. At first it recorded a pressure of 20 mm Hg. Then patient is asked to
116 | P a g e | By Dr. Manoj Kumar Poudel (Nepal- the country where Mt. Everest is present & the country
where Buddha was born.)

swallow food, then the pressure dropped to 0 mm Hg for short time, which then increased to
300 mm Hg jus for short time. After this the pressure in this site again came back to 20 mm
Hg. Based on these facts identify the portion of GIT where the manometer is:
A. Upper esophageal sphincter
B. Oesophageal body
C. Lower esophageal sphincter
D. Orad stomach
E. Caudad stomach
Answer C: Please observe the graph below to understand this:

151.
Following are the results of a patient:
Prothrombin time: 20 seconds
Partial thromboplastin time: 35-45 seconds
Bleeding time: 6 minutes
Platelet count: 300,000/mm3
117 | P a g e | By Dr. Manoj Kumar Poudel (Nepal- the country where Mt. Everest is present & the country
where Buddha was born.)

What is your diagnosis?


A. Vitamin K deficiency
B. DIC
C. Von Willebrand disease
D. Hemophila
E. Disease due to aspirin
F. Thrombocytopenia
G. Liver failure
H. Uremia
Answer A:

Warfarin: inhibits vitamin K dependent factor; reversal of warfarin can be done by vitamin K.
Heparin: is an anticoagulant that activates antithrombin III finally inactivates thrombin.
Aspirin: antiplatelet effect by inhibiting the production of thromboxane.
Uremia platelet dysfunction

152.
Granulosa cells do not produce estradiol from cholesterol because they do not have
an active:
A. 17-Hydroxylase
B. Aromatase
C. 5-alpha-Reductase
D. Sulfatase
E. Steroidogenic acute regulatory protein
Answer A: Explanation in the question number 153 chart. Granulosa cells produce
estradiol from testosterone by the help of an enzyme aromatase.

118 | P a g e | By Dr. Manoj Kumar Poudel (Nepal- the country where Mt. Everest is present & the country
where Buddha was born.)

119 | P a g e | By Dr. Manoj Kumar Poudel (Nepal- the country where Mt. Everest is present & the country
where Buddha was born.)

153.
This is a testicular steriod hormone secreted by Leydig cells. The immediate precursor
of this hormone is androstenedione during the synthesis. What is TRUE about this?
A. Bound to high-density lipoprotein (HDL)
B. Bound to activin
C. Converted to dihydrotestosterone in the prostate
D. Converted to 17-hydroxyprogesterone in the liver
E. Metabolized by cholesterol sidechain cleavage enzyme
Answer C: Activin has opposite function than that of inbihin. Inhibin inhibits synthesis
& secretion of FSH. Inhibin is secreted when there is higher level of FSH in plasma.

120 | P a g e | By Dr. Manoj Kumar Poudel (Nepal- the country where Mt. Everest is present & the country
where Buddha was born.)

154.
A 26-year old woman is brought to the emergency department because of a 4-day
history of flu-like symptoms accompanied by vomiting following each attempt to eat or drink.
Her temperature is 38.5C (101.3F), pulse is 93/min, respirations are 24/min, and blood
pressure is 105/70 mm Hg. Physical examination shows no other abnormalities. Which of the
following additional findings is most likely in this patient?
A. Decreased serum ADH (vasopressin) concentration
B. Increased serum aldosterone concentration
C. Increased serum atrial natriuretic peptide
D. Increased urine sodium and chloride concentrations
E. Increased urine volume
Answer B: This is a case of volume contraction increased aldosterone production
to retain sodium and water. Effects of aldosterone on sodium ions, potassium ions,
water and hydrogen ions are very important.
You need to know in which part of nephron aldosterone acts.
You need to know in which part of nephron ADH acts.
Angiotensin II vasoconstricts efferent arteriole more than afferent.
You need to know about macula densa and its feedback mechanism.
121 | P a g e | By Dr. Manoj Kumar Poudel (Nepal- the country where Mt. Everest is present & the country
where Buddha was born.)

Auto-regulation of blood-flow in kidneys, so blood flow is regulated in


denervated kidney as well (kidney transplantation)
Juxtraglomerular cells secrete renin in response to decreased renal blood
pressure.
RAAS: Dont miss that higher aldosterone level in blood inhibit further secretion
of renin.

155.
A 22-year-old man is brought to the emergency department because of a 6-hour
history of severe, sharp, upper back pain. He has had progressive fatigue during the past 3
weeks. He is 183 cm (6 ft) tall and weighs 79 kg (175 lb); BMI is 24 kg/m2. His temperature
is 36.9C (98.5F), pulse is 90/min, and blood pressure is 160/55 mm Hg. Physical
examination shows long, thin upper and lower extremities. Fingertip to fingertip with arms
outstretched is 189 cm (74 in) wide. A high-pitched midsystolic click is heard predominantly
over the apex. Which of the following best describes the primary genetic cause of this
patients condition?
A. Expression of genomic duplication within the fibrin gene
B. Mutation in keratin-14 gene
C. Nonsense mutation in fibrillin-1 gene
D. Overexpression of collagen X gene
E. Overexpression of fibronectin gene
Answer C: Memorise this answer for Marfan syndrome.

122 | P a g e | By Dr. Manoj Kumar Poudel (Nepal- the country where Mt. Everest is present & the country
where Buddha was born.)

In the mean please learn about osteogenesis imperfecta below.

Dont miss Ehler-Danlos syndrome if asked!: collagen, hyperflexibility


156.
A 55-year-old woman who is obese has a greater risk for endometrial carcinoma than
a 55-year-old woman with the same health history and status who is not obese. Which of the
following best explains this increased risk?
A. Accelerated catabolism of antioxidants
B. Association of obesity with smoking
C. Carcinogenic effects of dietary fats
D. Greater average number of pregnancies
E. Impairment of immune surveillance by T lymphocytes
F. Increased production of estrogen by adipose tissue
G. Later age of menopause
H. More frequent episodes of vaginitis
123 | P a g e | By Dr. Manoj Kumar Poudel (Nepal- the country where Mt. Everest is present & the country
where Buddha was born.)

Answer F: Adipose tissue has lipids, the precursor for steroid hormones. Please
memorise the table below. Please memorise following table, which hormone is
responsible for causing which cancer:

157.
A 35-year-old man has an adenoma of the parathyroid gland, with increased serum
concentrations of parathyroid hormone (PTH) and calcium. In this patient, PTH induces
which of the following processes to cause hypercalcemia?
A. Production of 25-hydroxycholecalciferol
B. Shift of Ca2+ from the intracellular to the extracellular fluid compartment
C. Stimulation of osteoclast activity
D. Suppression of renal production of 1,25-dihydroxycholecalciferol
Answer C:

124 | P a g e | By Dr. Manoj Kumar Poudel (Nepal- the country where Mt. Everest is present & the country
where Buddha was born.)

158.
A demonstration is performed during a lecture on muscle physiology in which a
student is asked to fully extend his right arm with the palm up. Two large textbooks are
placed on his palm, one at a time. Which of the following facilitates the maximum amount of
tension that allows the student to keep his arm extended in place under the increasing
weight of the books?
A. Amount of Ca2+ released from the sarcoplasmic reticulum
B. Amount of muscle phosphocreatine
C. Amplitude of the action potential
D. Number of motor units recruited
E. Rate of cross-bridge recycling
Answer D: direct answer question.
159.
Study the following ano-rectal manometry findings below. The arrow shows the time at
which the balloon in the rectum is inflated.

125 | P a g e | By Dr. Manoj Kumar Poudel (Nepal- the country where Mt. Everest is present & the country
where Buddha was born.)

Identify a single cause for the graphs represented by A?


A. Fecal incontinence
B. Obstipation
C. Damage of rectal mucosal
D. Local anesthesia in the perineal region
E. Hirshsprungs disease
Answer E: Due to neurological problem in this disease, even when a pressure is
generated in rectum, the reflex movement in distal part can not occur.

160.
During the hospital round in gastro-surgery inpatient ward of Saint Jude Hospital in
Vieux Fort, we come across Mr. Gonzalo, a 72-year-old-man from Pierott, who have had an
ileal resection (removal of the ileum). Miss. Tamico, a second semester medical student from
International American University, immediately thought about entero-hepatic circulation of
bile salts. Which of the following can most probably happen with Mr. Gonzalo due to the ileal
resection?
A. Constipation.
B. Increased peristalsis.
C. Decreased secretion of pancreatic lipase.
D. Derangement of the function of secretin.
E. Steatorrhoea.
Answer E:
126 | P a g e | By Dr. Manoj Kumar Poudel (Nepal- the country where Mt. Everest is present & the country
where Buddha was born.)

Vitamin B 12 gets absorbed from terminal ileum with the help of intrinsic factor.
161.
A 24-year-old male from Manchester is a professional soccer player. After the 90minute long game he was taking rest in his changing room. He notices that had been soaked
with sweat. Which of the following glands are responsible for the foul smell of this guy?
A. Eccrine
B. Merocrine
C. Apocrine
D. Holocrine
E. Paracrine
Answer C:

162.
Mr. Nick, 48-year-old patient has inflammation of duodenum. Which of the following
can happen in this patient from affluent family?
A. Decreased acid out-put
B. Perinicious anemia
C. Tetany
D. Dumping
127 | P a g e | By Dr. Manoj Kumar Poudel (Nepal- the country where Mt. Everest is present & the country
where Buddha was born.)

E. Renal stones
Answer C: Tetany is the feature of hypocalcemia. Dumping occurs in gastrectomy.
Renal stones occur in a patient with hyperclcemia, as calcium stones are the
commonest renal stones.

163.
After a meal of high-content lipids, dietary lipid is absorbed by the small intestine. And
it is transported in the lymph mainly as:
A. Free fatty acids bound to albumin
B. LDLs
C. HDLs
D. Chylomicrons
E. Micelles
Answer D: Never get confused in between micelles (for absorption) and
chylomicrons (for transport).

164.
Although total gastrectomy with distal pancreatectomy has been the standard surgical
treatment for advanced gastric cancer, this procedure is frequently complicated by leakage of
pancreatic juice and postoperative diabetes mellitus. We present the case of a 74-year-old
woman with proximal gastric cancer who underwent a laparoscopic modification of an open
pancreas-preserving total gastrectomy procedure first described by Maruyama et al. in 1995.
128 | P a g e | By Dr. Manoj Kumar Poudel (Nepal- the country where Mt. Everest is present & the country
where Buddha was born.)

With this novel technique, there is no pancreatic leakage, and at 12-month follow-up our
patient remains free of diabetes. Following total gastrectomy, on top of other medications, we
prescribed her vitamin B12 via .. ?
A. Oral route with special instruction to chew the tablet before swallowing.
B. Oral route with special instruction to take the medicine before eating the food.
C. Intramuscular or subcutaneous injection.
D. Topical application while exposing to sunlight.
E. Eating a lot of food daily which are rich in cobalamin; like pan-fired beef liver,
simmered turkey giblets, Braunschweiger pork liver sausage, raw Pacific oysters, etc.
Answer C: Please learn about R-protein, which is secreted by salivary glands. It binds
with Vit-B12 and protects it from gastric acid in stomach. When it reaches duodenum,
pancreatic protease breaks the complex of R-protein & vit B12 so that now vit B12 will
bind with intrinsic factor to get absorbed. If there is no stomach, there wont be
intrinsic factor (hence no absorption of vit B12), so the supplementation should be
given in injection form.

165.
Most of the following gastrointestinal secretions have a basal output during the
interdigestive period (between meals). However, the sight and smell of a tasty meal
stimulates gastrointestinal secretions. Of the various gastrointestinal secretions, which is the
most stimulated?
A. Intestinal secretion
B. Pancreatic secretion
129 | P a g e | By Dr. Manoj Kumar Poudel (Nepal- the country where Mt. Everest is present & the country
where Buddha was born.)

C. Gastric secretion
D. Biliary secretion
E. Salivary secretion
Answer E: You need to know this.

166.
Mr. Yesh, a 24-year-old college student was referred from orthopedic ward with his rib
fracture. He did not give any significant traumatic history for the rib fracture. He is now
admitted in endocrine unit of Department of Medicine, Tribhuvan University Teaching
hospital. The inference of lab reports are:
Serum PTH :
Serum calcium: 4.5 mmol/L
Serum phosphate: 2.5 mmol/L
Serum 1,25-dihydroxycholecalceferol:
Urine phosphate:
Urine calcium: 10 mmol/24 hour
Urine cAMP:
What would be your working diagnosis for this case?
A. Chronic renal failure
B. Surgical hypoparathyroidism
C. Pseudohypoparathyroidism
D. Humoral hypercalcemia of malignancy
E. Primary hyperparathyroidism
F. Osteomalacia
Answer E:

130 | P a g e | By Dr. Manoj Kumar Poudel (Nepal- the country where Mt. Everest is present & the country
where Buddha was born.)

Pseudohypoparathyroidism is a genetic disorder that is similar to


hypoparathyroidism; is caused by the body's lack of response to PTH instead of
reduced production of the hormone.
Parathyroid hormone-related protein (PTHrP) is primary cause of Humoral
Hypercalcemia of Malignancy and is overexpressed by many types of neoplasms.
167.
Miss. Tamika, a 24-year-old patient was diagnosed with systemic lupus
erythematosus and during the flaring of the symptoms his doctor prescribed her injection
methylprednisolone in hospital and changed it to oral prednisolone during her discharge,
where both drugs are corticosteroids. If she were to prescribe this drug for a long duration,
which of the following things the doctor would be worried about?
A. Increased chances of infection due to immunosuppression.
B. Exaggerate inflammatory response.
C. Increase insulin sensitivity.
D. Decrease vascular responsiveness to catecholamines.
E. Decrease gluconeogenesis and proteolysis.
Answer A: The chart below is very important to understand and remember.

131 | P a g e | By Dr. Manoj Kumar Poudel (Nepal- the country where Mt. Everest is present & the country
where Buddha was born.)

In males, adrenal androgens play only a minor role because de novo synthesis of
testosterone from cholesterol in the testes is much greater than testosterone
synthesis from adrenal androgenic precursors. In females, however, adrenal
androgens are the major androgens, and they are responsible for the development of
pubic and axillary hair and for libido.
168.
Diazoxide is a potassium channel activator. What happens to insulin secretion in cells of endocrine pancreas when diazoxide is prescribed for a 35-year-old bisexual patient
with altered level of glucose concentration in blood?
A. Increases
B. Decreases
C. No effect at all
D. It can either increase or decrease, which depends on the types of sex hormones he
has in his plasma.
E. It can either increase or decrease, which depends on 17,20-lyase in zona glomerulosa
of adrenal cortex.
Answer B: Diazoxide: potassium channel activator, which causes local relaxation in
smooth muscle by increasing membrane permeability to potassium ions. This
switches off voltage-gated calcium ion channels which inhibits the generation of an
action potential. Hence inhibits insulin secretion & can be used in insulinoma or
hyperinsulinism. The mechanism of insulin secretion from beta cells in pancreas is
shown below.

132 | P a g e | By Dr. Manoj Kumar Poudel (Nepal- the country where Mt. Everest is present & the country
where Buddha was born.)

169.
A chronic alcoholic person was admitted to hospital with haemorrhagic shock. He has
been vomiting blood since this morning. His wife reports that more than 1.5 liters of blood
was lost. So the ER physician did endoscopy and variceal bleeding from lower region of
esophagus was detected. So, SengstakenBlakemore tube was inserted and the bleeding
came into control. He was given IV fluids and the surgery team decided to go for
portosystemic shunting surgery. After surgery which of the following hormone levels would
you expect to increase in blood?
A. Prolactin
B. Growth hormone releasing hormone
C. Testosterone
D. Growth hormone
E. Insulin
Answer E: Insulin and glucagon gets metabolized exclusively in liver. This bypass
surgery prevents much of blood go to liver. Hence hormones like insulin and
glucoagon gets less chance for metabolism, thereby increasing their level in the
systemic circulation.
170.
Mrs. Cadwell, a 35-year-old female was diagnosed with a rare TRH-secreting tumor.
Among many other features, which of the following would most likely be her presentation?
A. Prognathism
B. Severe hypotonic polyuria
C. Genu varum
D. Galactorrhoea
E. Cold intolerance
133 | P a g e | By Dr. Manoj Kumar Poudel (Nepal- the country where Mt. Everest is present & the country
where Buddha was born.)

Answer D: Galactorrhoea = excessive secretion of breast milk by mammary glands.

171.
Mrs. Jordon, a 24-year-old married woman, living with her husband, is not able to get
pregnant since last 4 years after marriage though the couple wants to have a child. In the
infertility center, it was found out that one of the pituitary hormones is deficient in her blood
so that she is unable to ovulate. The deficient hormone would belong to which of the
following groups according to the chemical structure:
A. Glycoproteins
B. Amines (tyrosine)
C. Neuropeptides
D. Neurocrines
E. Long peptides
Answer A: You have to know which hormone falls in which category and their
precurcor (red in color) as shown below.

172.

Which of the following hormones has steroid-like mechanism of action?


A. Oxytocin
B. Glucagon on -cells of pancreas

134 | P a g e | By Dr. Manoj Kumar Poudel (Nepal- the country where Mt. Everest is present & the country
where Buddha was born.)

C.
D.
E.
F.
G.
H.
I.
J.

Glucagon in target tissue


Insulin
Parathyroid hormone
Calcitonin
Thyroxine
Follicle stimulating hormone
Corticotrophin releasing hormone
Atrial natriuretic peptide
Answer G:

173.

Secretion of oxytocin (uterine contraction & breast milk ejection) in increased by:
A. Orgasm
B. Cervical dilatation
C. Sight, sound or smell of infant

135 | P a g e | By Dr. Manoj Kumar Poudel (Nepal- the country where Mt. Everest is present & the country
where Buddha was born.)

D. Suckling
E. All of the above
Answer E: Direct question. Though you may not get all of above question in NBME
Shelf Exam. I gave it for you to learn all the 4 different factors for now..
174.
A 24-year-old boy attempted suicide after he came to know that his girlfriend has been
dating with his best friend from college. The half-filled bottle brought by his mother is an
insecticide and has malathion in it as the major ingredient. He is excessively salivating,
lacrimating, and vomiting. His pupils are constricted. And he looked very irritated. Which of
the following alterations in neurochemical transmission at brain cholinergic synapses is the
most likely result of this poisoning?
A. Increased synthesis of choline acetyltransferase
B. Accumulation of excessive acetylcholine outside the cholinergic neuron (in the
synaptic cleft)
C. Accumulation of choline outside the cholinergic neuron (in the synaptic cleft)
D. Down-regulation of postsynaptic cholinergic receptors
E. Competitive blockade of cholinergic receptors
Answer B: We find acetylcholine in muscarinic receptors, CNS, skeletal muscle and
ANS ganglion.

136 | P a g e | By Dr. Manoj Kumar Poudel (Nepal- the country where Mt. Everest is present & the country
where Buddha was born.)

175.
A patient presents with reduced facial expression, spontaneous movements (slower
than normal) that are revealed most clearly when walking, monotonous speech, an increase
in muscle tone in arms, and a rhythmic resting tremor (4-7Hz) in the fingers, including a pillrolling tremor. He also has festinating and shuffling gait. This disorder can be directly linked
to loss of:
A. Glutamatergic inputs from neocortex to the neostriatum
B. Gamma aminobutyric acid (GABA)-ergic input to the lateral (external) pallidal segment
C. Glutamatergic input to the medial (internal) pallidal segment
D. Dopaminergic inputs to the neostriatum
E. Cholinergic inputs to neostriatum
Answer D

In CNS GABA & glycine are inhibitatory; Glutamate is excitatory neurotransmitter.

137 | P a g e | By Dr. Manoj Kumar Poudel (Nepal- the country where Mt. Everest is present & the country
where Buddha was born.)

Concept of myelinated vs. Unmylinated neurons:


Thick diameter vs. thin diameter:
Schwan cells (PNS) vs. Oligodendrocyte (CNS) myelin synthesis. Optic nerve is
myelinated by oligodendrocytes.

138 | P a g e | By Dr. Manoj Kumar Poudel (Nepal- the country where Mt. Everest is present & the country
where Buddha was born.)

176.
At equilibrium the concentration of Na + is 145 mmol/L in the extracellular fluid and
14.5mmol/L in the intracellular fluid. The equilibrium potential across the membrane for Na +
at 98.6 F temperature is calculated to be:
A. 61.5 mV
B. -61.5 mV
C. 100 mV
D. -100 mV
E. 70 mV
F. -70 mV
Answer A: The formula is separate for negative ions and positive ions.

177.
It was found that enough amount of glucose is not entering the cell though the blood
glucose level and serum insulin level were within normal limits when measured for 4 different
times of a day for 3 days. The clinician suspected that there must be some problem with
signal transduction or insulin receptor in the cell. Which of the following is most likely to have
derangements in this patient?
A. Protein kinase A
B. Phospholipase C
C. Protein kinase C
D. Protein kinase G
139 | P a g e | By Dr. Manoj Kumar Poudel (Nepal- the country where Mt. Everest is present & the country
where Buddha was born.)

E. Protein phosphatase
Answer E:

178.
A population of cells are exposed to radiation and then examined by electron
microscopy. Ribosomes are found to be detached from the endoplasmic reticulum. This
structural change will cause loss of which of the following functions?
A. Synthesis of steroid hormones
B. Synthesis of cytosolic proteins
C. Drug detoxification
D. Synthesis of secretory proteins
140 | P a g e | By Dr. Manoj Kumar Poudel (Nepal- the country where Mt. Everest is present & the country
where Buddha was born.)

E. Endocytosis
F. ATP production
Answer D: The damaged structure is RER. Learn the basic functions of other cellular
organelles too.

179.
One of the benefits of human chorionic somatomammotropin (hCS) secretion in
mother during the pregnancy is:
A. A reduction of her plasma glucose concentrations.
B. The blockage of the development of diabetes mellitus in later life.
C. The increased availability of glucose to the fetus.
D. A reduction of pituitary function.
E. Increased appetite.
Answer C: Dont miss points in these explanations. Specially about hcS and the
index of placental well-being and fetal well-being.

180.
In order to compare the electrical and concentration force, they must be expressed in
the same unit. This can be achieved by:
A. Goldman equation
B. vant Hoff equation
C. Ficks law
141 | P a g e | By Dr. Manoj Kumar Poudel (Nepal- the country where Mt. Everest is present & the country
where Buddha was born.)

D. Nernst equation
E. Permeability coefficient
Answer D: direct question
Brief summary on ECG/EKG that could be test in NBME Shelf Exam:
Depolarisation/repolarization:
o Repolarization occurs from apex to base (opposite to the direction of
depolarization)
o Depolarization occurs from endocardium to epicardium (opposite to the
repolarization which is from epicardium to endocardium)
P wave peaking: Right atrial hypertrophy
Broad and bifid P wave: Left atrial hypertrophy
Ventricular hypertrophy: higher amplitude (height) of QRS & cardiac axis
deviation.
Cardiax axis: shifts to right in RV hypertrophy; shifts to left in LV hypertrophy.
RVH seen in V1; LVH seens in V6 the best.
QRS duration prolongation: bundle branch block, hyperkalemia
Q wave: septal depolarization (left to right)
QT inverval: represents first ventricular depolarization to last ventricular
repolarization;
ST segment: period of ventricular depolarisation, ( ST is portion of QT, which
represents plateau phase of ventricular action potential.)
ST elevation: recent MI or pericarditis: anterior infarction seen in V leads,
inferior infarction in lead II and VF; pericarditis has ST elevation everywhere.
Physiological U wave: represent repolarization of the papillary
muscles or Purkinje fibers.
Pathological U wave: hypokalemia, MI.
J-point: point at which the QRS complex finishes and the ST segment begins.
Saw-toothed ECG: atrial flutter
WPW syndrome: short PR interval and slurring of the QRS upstroke suggests
Torsades de pointes: form of polymorphic ventricular tachycardia (VT)
characterized by a gradual change in the amplitude and twisting of the QRS
complexes around the isoelectric line. The morphology of the QRS complexes
varies from beat to beat. The ventricular rate can range from 150 beats per
minute (bpm) to 250 bpm. And QT interval be increased markedly.
ECG progression of myocardial infarction: 1st sign of myocardial infarction is
hyperacute T wave (2-30 minutes duration) ST segment elevation
Pathological Q waves may appear within hours or may take greater than 24 hr.
The T wave will generally become inverted in the first 24 hours, as the ST
elevation begins to resolve.
T wave inversion: MI, ventricular hypertrophy, bundle branch block, digoxin
toxicity and can be normally seen in some leads.
142 | P a g e | By Dr. Manoj Kumar Poudel (Nepal- the country where Mt. Everest is present & the country
where Buddha was born.)

Dextrocardia: Right axis deviation, positive QRS complexes in aVR, Lead I


showed global negativity.

RBBB: wide QRS complex, RSR pattern in V1 and deep wide S wave in V6
LBBB: wide QRS complex, M pattern in QRS complex specially in I, VL, V5 & V6; inverted
T waves in I, II & VL
Mneumonics: MARROW & WILLIAM:

LOX & COX pathyway. Mechanism of action of steroid and NSAIDs:

143 | P a g e | By Dr. Manoj Kumar Poudel (Nepal- the country where Mt. Everest is present & the country
where Buddha was born.)

Orlistat inhibits pancreatic lipase & decreases fat absorption.


Sibutramine is norepinephrine, dopamine, and serotonin reuptake inhibitor.
144 | P a g e | By Dr. Manoj Kumar Poudel (Nepal- the country where Mt. Everest is present & the country
where Buddha was born.)

145 | P a g e | By Dr. Manoj Kumar Poudel (Nepal- the country where Mt. Everest is present & the country
where Buddha was born.)

146 | P a g e | By Dr. Manoj Kumar Poudel (Nepal- the country where Mt. Everest is present & the country
where Buddha was born.)

147 | P a g e | By Dr. Manoj Kumar Poudel (Nepal- the country where Mt. Everest is present & the country
where Buddha was born.)

148 | P a g e | By Dr. Manoj Kumar Poudel (Nepal- the country where Mt. Everest is present & the country
where Buddha was born.)

The following normal physiological feedback control of leptin is lost in OBESITY:

THE END
BEST WISHES
149 | P a g e | By Dr. Manoj Kumar Poudel (Nepal- the country where Mt. Everest is present & the country
where Buddha was born.)

Vous aimerez peut-être aussi